SlideShare a Scribd company logo
1 of 351
SEGUNDO
SIMULADOR DEL CURSO
ENARM CMN SIGLO XXI
PRIMERA PARTE
Instrucciones
1. Preparar hoja de respuesta impresa.
2. Activar el simulador aplicando play.
3. Escribir nombre y hora de inicio en la hoja de
respuesta.
4. Circular el reactivo de cada respuesta en hoja
de respuesta.
5. El simulador se activará a las 8:00 hrs y se
desactivará a las 14:00 hrs, la primera parte.
6. La segunda parte se activara a las 16:00 hrs y de
desactivará a las 20:00 hrs.
Caso clínico No. 58 (ejemplo)
Femenino de 51 años, originaria de
Campeche, acude control mensual por HAS y DM
10 y 5 años respectivamente, sus SV fueron TA
155/95 mmHg, FC 51 lpm, FR 21 rpm. Glucosa
210 mg/dl, hemoglobina glucosilada 6, urea
17, creatinina 1,1 mg/dl, refiere mareo
ocasional, cefalea global acompañada de sueño y
fatiga, frio y mareo ocasional, disnea de leves a
moderados esfuerzos, se observa edema de
miembros inferiores. Su tratamiento es
captopril, metoprolol y glibenclamida, se realiza
ECG (ver imagen).
Caso clínico No. 58 (ejemplo)
Caso clínico No. 58 (ejemplo)
PRENGUNTA
Se realiza ECG, considerando las manifestaciones que se
han presentado, que conducta terapéutica considera
más apropiada realizar de forma mediata?
RESPUESTA
a.- Incrementar los fármacos hasta meta de <120/80
mmHg.
b.- Retirar metoprolol continuar con captopril y
glibenclamida.
c.- Mantener glibenclamida, retirar metoprolol e iniciar
losartan.
d.- Agregar hidroclorotiazida al manejo actual
Hoja de respuestas
d 51 a b c d 101
d 52 a b c d 102
d 53 a b c d 103
d 54 a b c d 104
d 55 a b c d 105
d 56 a b c d 106
d 57 a b c d 107
d 58 a b c d 108
d 59 a b c d 109
d 60 a b c d 110
INICIO
BUENA SUERTE
CASE REPORT
A recently married 29-year-old nulliparous African-
American female presents with uncomplicated
cystitis. She is otherwise healthy. She reports that
she is currently using barrier birth control methods
but plans to discontinue this soon, as she would
like to become pregnant.
QUESTION No. 1
Which one of the following supplemental vitamins or
minerals would you advise this patient to
begin taking?
a) Thiamine
b) Iodine
c) Folate
d) Calcium
QUESTION No. 2
Which one of the following is true regarding testing
the rotator cuff for injury?
ANSWER
a) The subscapularis is tested with lateral rotation
against resistance
b) The teres minor is tested with medial rotation
against resistance
c) The infraspinatus is tested with medial rotation
against resistance
d) The supraspinatus is tested with abduction
against resistance
CASE REPORT
You have recently begun caring for a 25-year-old white
female who has multiple complaints. You have seen
her 3 times for walk-in office visits over the past
month. She has shown appreciation
for your work during the encounters, but has been
critical of your care when talking to the office
staff. At times she has been kind and charming, and at
other times she has been rude and verbally
abusive to your staff. She has a string of multiple
relationships in the past, none of which has
lasted very long. During times of intense stress, she has
sometimes engaged in self-mutilation.
She frequently changes jobs and living arrangements.
QUESTION No. 3
Which one of the following strategies would be most
appropriate in the care of this patient
ANSWER
a) Strive to develop a close relationship with the
patient.
b) Ignore verbal attacks on staff members.
c) Provide detailed, technical explanations for. any
therapies provided.
d) Schedule frequent office visits for follow-up.
In an 11-year-old male with dark brown urine and
hand and foot edema.
QUESTION No. 4
Which one of the following would be most suggestive
of glomerulonephritis?
ANSWER
a) RBC casts in the urine
b) Elevated C3 and C4 complement levels
c) Eosinophils in the urine
d) Positive serum antinuclear antibody levels
A 32-year-old white female at 16 weeks gestation
presents to your office with right lower quadrant pain.
QUESTION No. 5
Which one of the following imaging studies would be
most appropriate for initial evaluation of this patient?
ANSWER
a) MRI of the Abdomen
b) Intravenous pyelography
c) CT of the abdomen
d) Ultrasonography of the abdomen
QUESTION No. 6
Which one of the following is the leading cause of
blindness in individuals over age 65?
a) Narrow angle glaucoma
b) Diabetic retinopathy
c) Macular degeneration
d) Ophthalmic artery occlusion
CASE REPORT
A 48-year-old white female complains of anxiety and
difficulty concentrating at home and at work. She
reports that the symptoms have increased over the
last 2 months because of her daughter’s marital
difficulties. She has had similar symptoms along with
intermittent depression since she was a teenager.
She admits to a loss of pleasure in work and
recreational activities.
QUESTION No. 7
Which one of the following is LEAST likely to help her
coexistent depressive symptoms?
ANSWER
a) Nortriptyline.
b) Buspirone.
c) Escitalopram
d) Venlafaxine
CASE REPORT
An 82-year-old white female visits your office. She reports a 2-
week history of shoulder stiffness that is severe in the
morning, and a 3-day history of pain on the left side of the
face while chewing food. She denies dental pain or
sensitivity. Her family history is negative for rheumatologic
disorders. Your evaluation reveals a 5-lb weight loss and
evidence of mild depression. Her temperature is 38.0° C
(100.4° F), she has no dental decay or carotid bruit, and her
left temporal scalp is tender. Her lungs are clear to
auscultation, there are no abnormal heart sounds, and her
abdomen is nontender. She has no joint swelling or
warmth, but experiences discomfort with shoulder and hip
range of motion. Laboratory analysis reveals a hemoglobin
level of 11.0 g/dL (N 11.7–16.1), an erythrocyte
sedimentation rate of 80 mm/hr (N 0–30), a serum glucose
level of 120 mg/dL, and a BUN level of 24 mg/dL (N 8–23).
QUESTION No. 8
Which one of the following is the greatest immediate
risk for this patient?
ANSWER
a) Monocular vision loss
b) Hemiparesis
c) Sudden death
d) Adhesive capsulitis of the shoulder
CASE REPORT
A 73-year-old male with COPD presents to the
emergency department with increasing dyspnea.
Examination reveals no sign of jugular venous
distention. A chest examination reveals decreased
breath sounds and scattered rhonchi, and the heart
sounds are very distant but no gallop or murmur
is noted. There is +1 edema of the lower extremities.
Chest radiographs reveal cardiomegaly but
no pleural effusion. The patient’s B-type natriuretic
peptide level is 850 pg/mL (N <100) and his
serum creatinine level is 0.8 mg/dL (N 0.6–1.5).
QUESTION No. 9
Which one of the following would be the most
appropriate initial management?
ANSWER
a) Prednisone, 20 mg twice daily for 1 week
b) Furosemide, 40 mg intravenously
c) Intravenous heparin
d) Tiotropium
CASE REPORT
A 28-year-old primigravida is at 20 weeks gestation by
dates but her fundal height is consistent with a 26-
week gestation. She has had episodes of vomiting
during the pregnancy that were more severe than
the physiologic vomiting typically seen in pregnancy.
A sonogram performed at about 5 weeks gestation
for vaginal bleeding was normal and showed a single
fetus.
QUESTION No. 10
Which one of the following would be most
appropriate at this point?
ANSWER
a) Expectant management
b) A repeat sonogram
c) MRI of the pelvis
d) A serum hCG leve
CASE REPORT
A 24-year-old male, new to your practice, presents for a
mental health evaluation. The patient has a past history
of schizophrenia, diagnosed several years ago.
QUESTION No. 11
Which one of the following, if present, would lead to a
reconsideration of this diagnosis?
ANSWER
a) Loose associations
b) Elated mood
c) Social dysfunction
d) Auditory hallucinations
CASE REPORT
A previously healthy 22-year-old female presents for
her regular prenatal checkup at 38 weeks gestation.
She has a blood pressure of 145/95 mm Hg today
and this is unchanged 1 hour later. Her blood
pressure was normal before pregnancy. She is
otherwise feeling well. She has moderate edema at
the ankles and 3+ reflexes at the knees and ankles.
A urinalysis for protein is normal.
QUESTION No. 12
Given this presentation, which one of the following
is the most likely diagnosis?
ANSWER
a) Hemolysis, elevated liver enzymes, low platelets
syndrome
b) Preeclampsia
c) Essential hypertension
d) Gestational hypertension
CASE REPORT
A 72-year-old female with longstanding diabetes
mellitus presents to your office. During the review
of systems, she complains of difficulty voiding and
frequent “dribbling.” A urinalysis is negative for
infection and her post-void residual volume is 250
mL.
QUESTION No. 13
Which one of the following is the most likely cause of
this patient’s urinary incontinence?
ANSWER
a) A grade II cystocele
b) Atrophic vaginitis
c) Asymptomatic bacteriuria
d) Autonomic neuropathy
CASE REPORT
A 3-year-old male is brought to your office the day
after he was stung by a honeybee. He has
developed a significant local reaction, with redness
and swelling around the site of the sting on his
forearm, and also has had some swelling of his lips
for “a couple of hours.” His mother removed the
stinger and gave him some oral diphenhydramine
The local reaction has now almost resolved and he
has not had any hives or respiratory distress.
QUESTION No. 14
Which one of the following is true concerning this
situation?
ANSWER
a) An antibiotic that covers Streptococcus pyogenes
and Streptococcus aureus should be administered
b) If venom immunotherapy is begun, it should be
stopped after 2 years of treatment
c) The child is at high risk for a systemic reaction if he
is stung again in the future
d) Skin tests are not helpful in confirming the
presence of insect sting allergy
CASE REPORT
A 65-year-old white female develops a burning pain in the
left lateral thorax, followed 2 days later by an
erythematous vesicular rash.
QUESTION No. 15
Of the following, the best treatment is?
ANSWER
a) topical capsaicin
b) oral corticosteroids
c) topical corticosteroids
d) topical acyclovir
QUESTION No. 16
Which one of the following would be most
appropriate to treat a dental infection requiring
antibiotic therapy?
ANSWER
a) Penicillin
b) Dicloxacillin
c) Erythromycin
d) Tetracycline
CASE REPORT
You are consulted for medical management of a 45-
year-old male, previously unknown to you,
who is hospitalized in the psychiatric unit with
paranoid schizophrenia. His fasting blood glucose
level is 180 mg/dL.
QUESTION 17
Which one of the following medications is the most
likely cause of the hyperglycemia?
ANSWER
a) Alprazolam
b) Chlorpromazine
c) Olanzapine
d) Haloperidol
CASE REPORT
A 22-year-old male presents to the emergency
department several hours after a rugby match in
which he was struck in the face. His nose is bleeding.
On examination the patient has a depression in his
nose on the side of impact and an outward
displacement on the opposite side. Internal
inspection reveals a purple area of swelling and
fluctuance on the left side of the nasal septum. The
remainder of the examination demonstrates no
findings to suggest other facial or periorbital trauma.
QUESTION 18
Which one of the following would be most appropriate
at this time?
ANSWER
a) Immediate CT imaging to assess for facial and
mandibular fracture
b) Incision and drainage of the swollen, fluctuant area
c) Immediate plain radiographs of the nose and orbits
d) Immediate closed reduction of the nasal fracture
CASE REPORT
A 30-year-old gravida 3 para 2 at 28 weeks gestation is
a restrained passenger in a high-speed motor
vehicle accident. After initial stabilization in the field
with supplemental oxygen and intravenous
fluids, she is brought into the emergency
department on a backboard and wearing a cervical
collar.
CASE REPORT
Your hospital administrator asks you to develop a
community screening program for melanoma.
QUESTION 19
Which one of the following is true concerning screening for
this disease?
ANSWER
a) Screening for melanoma is not indicated since screening
takes too much time
b) No definite clinical evidence has shown that screening
for melanoma reduces mortality
c) Screening for melanoma is not indicated since the
disease is rare
d) Because of sunbathing, female patients are the most
important population to screen
CASE REPORT
A 19-year-old white female presents for an initial family
planning evaluation. Specifically, she is interested in
oral contraception. She is not presently sexually
active, but has a steady boyfriend. She has no
contraindications to oral contraceptive use. She has
mild acne vulgaris. You discuss possible side effects
and benefits of combined oral
contraceptives, including improvement of her acne.
QUESTION 20
Which one of the following is also associated with oral
contraceptive use?
ANSWER
a) Increased incidence of dysmenorrhea
b) Increased risk of ovarian cancer
c) Increased risk for ectopic pregnancy
d) Decreased risk of ovarian cysts
CASE REPORT
A 78-year-old white female notices a scant milky
secretion from her breast.
QUESTION 21
Which one of the following medications can cause
this symptom?
ANSWER
a) Diazepam
b) Risperidone
c) Famotidine
d) Zaleplon
CASE REPORT
You have just diagnosed and treated gonorrheal
cervicitis in a 24-year-old female who is in
her second trimester of pregnancy. The
patient has a friend who had a stillborn infant
and she is concerned that the gonorrhea may
predispose her to stillbirth.
QUESTION 22
Which one of the following would be most appropriate in
this situation?
ANSWER
a) Perform monthly vaginal cultures for
gonorrhea, starting at 24 weeks gestation
b) Perform fetal monitoring and serial vaginal
cultures, starting at 32 weeks gestation
c) Inform the patient that stillbirth related to gonorrhea
is very rare, and that special fetal
and maternal monitoring is not needed
d) Administer prophylactic ciprofloxacin (Cipro) weekly
until delivery
CASE REPORT
A 6-year-old male is brought in for evaluation by his
mother, who is concerned that he may have
asthma. She reports that he coughs about 3 days
out of the week and has a nighttime cough
approximately 1 night per week. There is a family
history of eczema and allergic rhinitis.
QUESTION 23
Which one of the following would be the preferred
initial treatment for this patient?
ANSWER
a) A mast-cell stabilizer such as cromolyn sodium
b) A leukotriene receptor antagonist such as
montelukast
c) A long-acting beta-agonist such as salmeterol
d) A low-dose inhaled corticosteroid such as
budesonide
QUESTION 24
Which one of the following is the most correct recommendation
regarding seat belt use for a woman at 38 weeks gestation?
ANSWER
a) The seat belt should be positioned over the dome of the uterus
and the shoulder harness should be positioned between the
breasts
b) The seat belt should be positioned under the abdomen over both
the anterior superioriliac spines and the pubic symphysis; the belt
should be applied with some slack
c) The seat belt should be positioned under the abdomen over both
the anterior superior iliac spines and the pubic symphysis; the
shoulder harness should be positioned between the breasts; the
belt should be applied as snugly as comfort will allow
d) Seat belts should not be used in the later stages of pregnancy
CASE REPORT
A 34-year-old female presents to the emergency
department with a severe migraine headache
unresponsive to tramadol and sumatriptan at home.
She takes fluoxetine for depression. Soon after being
given an injection of meperidine, she develops
agitation, diaphoresis, tremor, diarrhea, fever, and
incoordination.
QUESTION 25
The most likely cause of this patient’s symptoms is?
ANSWER
a) serotonin syndrome
b) panic attack
c) thyrotoxic storm
d) viral encephalitis
QUESTION 26
The development of azotemia suggests the presence
of underlying renal artery stenosis in a patient
taking which one of the following?
ANSWER
a) Amlodipine
b) Lisinopril
c) Hydrochlorothiazide
d) Metoprolol
QUESTION 27
Which one of the following is a risk factor for
endometrial cancer?
ANSWER
a) Multiparity
b) Use of oral contraceptives
c) Late menarche
d) Polycystic ovary syndrome
QUESTION 28
Promoting good sleep hygiene is basic in the treatment of
insomnia. Which one of the following measures will aid
in promoting healthy sleep habits?
ANSWER
a) Taking an enjoyable book or magazine to bed to read
b) Eating the heaviest meal of the day close to bedtime
c) Drinking a glass of wine as a sedative before retiring
d) Maintaining a regular sleep/wake schedule
QUESTION 29
Until you are able to rule out a spinal injury, in what
position should the patient be kept?
ANSWER
a) Supine, with the uterus manually deflected
laterally
b) Prone
c) Trendelenburg’s pattern
d) Left lateral decubitus
QUESTION
A 47-year-old male presents with a history of
fatigue, arthralgias, nonspecific abdominal
pain, and erectile dysfunction. The initial
laboratory workup reveals a normal CBC and
basic metabolic profile, but slightly elevated
transaminases.
QUESTION 30
Which one of the following is the most appropriate
initial test to evaluate for hereditary
hemochromatosis?
ANSWER
a) Serum transferrin saturation
b) Serum ceruloplasmin testing
c) Serum alpha-fetoprotein testing
d) Serum ferritin testing
CASE REPORT
A 56-year-old female presents for a health
maintenance examination. She has a history of a
total hysterectomy for benign disease 4 years ago.
You are able to document that the hysterectomy
pathology was benign and that she has had normal
Papanicolaou (Pap) tests for 10 years. The patient
asks about regular Pap smears.
QUESTION 31
Which one of the following would be the most
appropriate recommendation?
ANSWER
a) Routine Pap smears should be continued until age
70
b) A Pap smear should be done every 3 years
c) A Pap smear should be done yearly for 3 years and
only if indicated thereafter
d) A Pap smear is not indicated
QUESTION
A 31-year-old African-American female presents with
the chief complaint of bilateral galactorrhea of 3
months’ duration. She also has not menstruated for
1 year despite changing birth control pills several
times. A review of systems is otherwise
noncontributory. Except for a milky discharge with
stimulation of the breasts, her examination is within
normal limits. Serum prolactin on two occasions is
>200 :g/L (N 0–20).
QUESTION 32
Which one of the following would be most
appropriate at this point?
ANSWER
a) Stop her oral contraceptive and repeat the serum
prolactin level in 1 month
b) Start the patient on risperidone
c) Order bilateral mammography
d) Order a brain MRI with enhancement and
emphasis on the pituitary fossa
CASE REPORT
An 84-year-old African-American female is brought to
your office by her daughter, who is concerned that the
mother has memory problems and is neglecting to pay
her monthly bills. The mother also is forgetting
appointments and asks the same questions repeatedly.
This problem has been steadily worsening over the last
1–2 years. The patient has very little insight into her
problems, scores 24 out of a possible 30 points on the
Mini-Mental State Examination, and has difficulty with
short-term recall and visuospatial tasks. Her physical
examination and a thorough laboratory workup are
normal. A CT scan of the brain mreveals diffuse
atrophy.
QUESTION 33
Which one of the following is the most likely etiology
for this patient’s memory problem?
ANSWER
a) Alzheimer’s disease
b) Dementia resulting from depression
c) Normal aging
d) Lewy body dementia
CASE REPORT
A 23-year-old female is at 8 weeks gestation with her
first pregnancy. She is planning to travel to Third
World countries soon for job-related reasons.
QUESTION 34
Which one of the following is contraindicated in this
patient?
ANSWER
a) Hepatitis B vaccine
b) Rabies vaccine
c) Varicella vaccine
d) Meningococcal vaccine
QUESTION 35
In the evaluation of foot ulcerations, a neuropathic
etiology is suggested by which one of the following?
ANSWER
a) Abnormal monofilament testing
b) Absence of toe hair
c) Erectile dysfunction
d) Distal foot pain when supine
QUESTION 36
A 72-year-old white male has new-onset hypertension
with a current blood pressure of 190/110mm Hg.
Which one of the following agents can be used as
part of a test for diagnosing renovascular
hypertension, but would also increase the risk for
azotemia if used for treatment?
ANSWER
a) Furosemide
b) Metoprolol
c) Captopril
d) Amlodipine
QUESTION 37
Which one of the following treatments for childhood
nocturnal enuresis has both the highest cure
rates and the lowest relapse rates?
ANSWER
a) Positive reinforcement
b) Imipramine
c) Responsibility training
d) Bed-wetting alarms
CASE REPORT
You see a 30-year-old male who has just fallen on an
outstretched hand. He complains of wrist pain and
edema. Examination reveals tenderness over the
anatomic snuffbox and over the scaphoid tubercle at
the proximal wrist crease with the hand in extension.
Radiographs of the wrist are negative.
QUESTION 38
Which one of the following would be the most
appropriate action at this point?
ANSWER
a) Order high-spatial-resolution ultrasonography of
the wrist
b) Order a bone scan for the next day
c) Immobilize in a thumb spica splint for 1–2 weeks
and then order repeat radiographs
d) Immobilize in a cast for 6–8 weeks
CASE REPORT
A 15-month-old male is brought to the emergency
department following a generalized tonic-clonic
seizure at home. The parents report that the
seizure lasted 5 minutes, with confusion for the
next 15 minutes. This is the child’s first seizure.
There is no family history of seizures. His medical
history is normal except for a 1-day history of a
URI. While initially lethargic in the emergency
department, the child is now awake and
playful, with a temperature of 39.5° C (103.2° F)
and a normal examination. Appropriate
diagnostic tests are performed, including a blood
glucose level, which is 96 mg/dL.
QUESTION 39
Which one of the following would be most appropriate
to administer at this point?
ANSWER
a) Ceftriaxone intravenously
b) Phenobarbital orally
c) Carbamazepine orally
d) Acetaminophen orally
CASE REPORT
A 77-year-old white male complains of urinary
incontinence of more than one year’s duration. The
incontinence occurs with sudden urgency. No
association with coughing or positional change has
been noted. There is no history of fever or dysuria.
One year ago he underwent transurethral resection
of the prostate (TURP) for benign prostatic
hypertrophy and says his urinary stream has
improved. Rectal examination reveals a smoothly
enlarged prostate without nodularity, and normal
sphincter tone. No residual urine is found with post-
void catheterization.
QUESTION 40
Which one of the following is the most likely cause of
this patient’s incontinence?
ANSWER
a) Overflow
b) Urinary tract infection
c) Fecal impaction
d) Detrusor instability
QUESTION 41
A health-care worker has a negative tuberculin skin test
(Mantoux method). A second test 10 days
later is positive. This result indicates
ANSWER
a) a false-positive skin test
b) long-standing, latent infection
c) probable immunodeficiency
d) previous vaccination with BCG
QUESTION 42
The most common manifestation of uterine rupture
during labor is
ANSWER
a) vaginal hemorrhage
b) regression of the fetus
c) sudden, tearing uterine pain
d) fetal distress
QUESTION 43
Which one of the following tests is most useful for the
initial workup of suspected Alzheimer’s disease?
ANSWER
a) Carotid Doppler ultrasonography
b) An erythrocyte
sedimentation rate
c) A cardiac stress test
d) A TSH level
CASE REPORT
A 78-year-old white female presents with a 3-day history of
lower thoracic back pain. She denies any antecedent fall or
trauma, and first noted the pain upon arising. Her
description of the pain indicates that it is
severe, bilateral, and without radiation to the arms or legs.
Her past medical history is positive for hypertension and
controlled diabetes mellitus. Her medications include
hydrochlorothiazide, enalapril, metformin, and a general
multivitamin. She is a previous smoker but does not drink
alcohol. She underwent menopause at age 50 and took
estrogen for “a few months” for hot flashes. Physical
examination reveals her to be in moderate pain with a
somewhat stooped posture and mild tenderness over T12–
L1. She has negative straight-leg raising and normal lower
extremity sensation, strength, and reflexes.
QUESTION 44
Which one of the following is true regarding this patient’s
likely condition?
ANSWER
a) Subcutaneous or intranasal calcitonin may be very
helpful for pain relief
b) Investigation for an underlying malignancy is
indicated
c) Prolonged (approximately 2 weeks) bed rest will
increase the chance of complete
recovery
d) An MRI or nuclear medicine bone scan should be
performed
QUESTION 45
Which one of the following agents used for tocolysis
has the unique adverse effect of respiratory
depression?
ANSWER
a) Ritodrine
b) Magnesium sulfate
c) Terbutaline
d) Indomethacin
QUESTION 46
A 5-year-old male is scheduled for elective hernia
repair at 11:00 a.m. Which one of the following
would be the most appropriate recommendation?
ANSWER
a) No solid food for 8 hours prior to surgery and clear
liquids until 2 hours prior to surgery
b) No solid food after midnight and nothing by mouth
8 hours prior to surgery
c) Nothing by mouth 8 hours prior to surgery
d) Nothing by mouth 2 hours prior to surgery
QUESTION 47
The only nonsexual behavior that is consistently
and strongly correlated with cervical dysplasia
and cervical cancer is
ANSWER
a) caffeine consumption
b) cigarette smoking
c) cocaine use
d) a high-fat diet
QUESTION 48
Which one of the following is the first-line antibiotic
treatment for uncomplicated acute otitis media?
ANSWER
a) Ceftriaxone
b) Trimethoprim/sulfamethoxazole
c) Amoxicillin
d) Cefuroxime
CASE REPORT
A 4-year-old male has a fever of 1 week’s duration. It
has been at or slightly above 38° C (101°F) and has
responded poorly to antipyretics. The patient
complains of photophobia, burning in his eyes, and a
sore throat. His mother also notes that his eyes look
red, his lips are red and cracked, and he has a
“strawberry tongue.” The child’s palms and soles are
erythematous and the periungual regions show
desquamation of the skin. He has minimally painful
nodes located in the anterior cervical region, about
2×2 cm in size. A Streptococcus screen is negative.
QUESTION 49
The most appropriate management at this time
would be?
ANSWER
a) intravenous nafcillin
b) intramuscular benzathine penicillin G 600,000 U
c) intravenous immune globulin and aspirin
d) prednisone, 2–3 mg/kg daily
QUESTION 50
Patients with symptomatic congestive heart failure
associated with a reduced systolic ejection
fraction or left ventricular remodeling should be
initially treated with which one of the following
agents?
ANSWER
a) Verapamil
b) An ACE inhibitor
c) Hydralazine
d) Warfarin
CASE REPORT
A 6-year-old white male visits your office with chief
complaints of a recent onset of fever, bilateral knee
and ankle pain, colicky abdominal pain, and rash. On
examination, his temperature is 38.3° C (101.0°
F), and there is a prominent palpable reddish-brown
rash on the buttocks and thighs. There is pain on
motion of his knees and ankles, and mild diffuse
abdominal tenderness. The stool is positive for occult
blood. Laboratory Findings: Hemoglobin 11.0 g/dL
Hematocrit 33% WBCs 14,500/mm; 85% segs, 3 15%
lymphs, Platelets 345,000/mm 3 Prothrombin time
12 sec
QUESTION 51
Which one of the following is the most likely diagnosis?
ANSWER
a) Henoch-Schönlein purpura
b) Rocky Mountain spotted fever
c) Acute iron ingestion
d) Systemic onset juvenile rheumatoid arthritis
QUESTION 52
Which one of the following drug classes is preferred for
treating hypertension in patients who also have
diabetes mellitus?
ANSWER
a) alpha-Blocking agents
b) ACE inhibitors
c) Centrally-acting sympatholytics
d) Calcium channel blockers
CASE REPORT
A 76-year-old white male with a history of recurrent
depression has recently become more depressed and
developed psychotic features. His symptoms have
not responded to antidepressants and antipsychotic
agents, prescribed by his psychiatrist. The
psychiatrist has recommended electroconvulsive
therapy (ECT) for the patient. The patient’s family
visits you to ask for your opinion and
recommendations regarding ECT in this individual.
QUESTION 53
In your consultation with this family, which one of the
following would be accurate advice regarding ECT?
ANSWER
a) It has a low response rate
b) It is efficacious and safe
c) There is evidence that it predisposes to the
development of dementia
d) It causes irreversible short-term memory loss
QUESTION 54
Metformin, which is normally used in the management
of diabetes mellitus, has also been shown to have a
beneficial effect in
ANSWER
a) hyperthyroidism
b) polycystic ovary syndrome
c) right ventricular hypertrophy
d) osteoporosis
CASE REPORT
A 45-year-old white male presents with a 3-day history
of new-onset acute upper abdominal pain radiating
to the back. His pulse rate is 110 beats/min and
other vital signs are normal. Bowel sounds are
hypoactive and epigastric tenderness without
guarding is present.
QUESTION 55
Which one of the following is most accurate regarding
laboratory testing for possible pancreatitis in this situation?
ANSWER
a) Serum amylase and lipase levels may be falsely low in renal
failure
b) Elevated serum triglyceride levels can cause falsely elevated
serum amylase levels
c) There is a direct correlation between serum amylase levels
and the severity of the
pancreatitis
d) A serum amylase level that is three times normal is highly
specific for pancreatitis
QUESTION 56
You are evaluating a 28-year-old primigravida for an
abnormal Papanicolaou (Pap) test. Which one of the
following procedures would be contraindicated?
ANSWER
a) Endocervical curettage
b) Human papillomavirus (HPV) testing
c) Colposcopy
d) Cervical staining
QUESTION 57
Which one of the following should be withheld before
the administration of intravenous contrast?
ANSWER
a) Insulin
b) Metformin
c) Rosiglitazone
d) Glyburide
CASE REPORT
You are considering how useful a new treatment might
be in preventing stroke. A well designed study is
reported with 200 patients in the treated group and
200 patients in the untreated group. The study finds
a 5-year risk of stroke of 3% in the treated group
versus 5% in the untreated group.
QUESTION 58
Assuming this study is valid and applicable to your
patient population, how many patients would you
have to treat for 5 years to prevent one stroke
(number needed to treat, or NNT)?
ANSWER
a) 200
b) 25
c) 400
d) 50
CASE REPORT
Over the past year, a 27-year-old female has had
marked feelings of anxiety, tension, and irritability
during the week preceding most menstrual
cycles, accompanied by extreme fatigue and
insomnia. She has regularly missed several days of
work each month because of fatigue. She has no
previous history of any health or mental
problems, and within a few days of the onset of her
period she is back to normal.
QUESTION 59
Which one of the following is true concerning this
condition?
ANSWER
a) Alprazolam is an effective first-line agent for
treatment of this condition
b) This condition is a variation of a depressive disorder
c) Neither biologic nor psychological factors play a part
in this condition
d) This problem can be effectively treated with
serotonergic antidepressants
CASE REPORT
An 81-year-old retired electrical engineer whose wife is a
diabetic was experimenting with his wife’s glucose meter
and found that his glucose level was 198 mg/dL. He used
her strips and lancets, and started his own log. After a
week, he brings the log to you. His premeal glucose levels
range from about 150 mg/dL to 250 mg/dL. A review of
his medications shows none that would be likely to
increase his glucose level. A physical examination does
not suggest glucose intolerance secondary to a process
other than diabetes. His hemoglobin A1c is 9.0%.
Additional laboratory studies should be performed
before prescribing
QUESTION 60
which one of the following for this patient?
ANWSER
a) Metformin
b) Glimepiride
c) Miglitol
d) Insulin NPH
QUESTION 61
A 23-year-old gravida 3 para 1 at 28 weeks gestation whose
blood type is O-negative is antibody positive (D antibody)
on a routine 28-week screen. Which one of the following
best describes the clinical significance of this finding?
ANSWER
a) The fetus HAS hemolytic disease and requires
appropriate monitoring and treatment
b) The current fetus is NOT at risk for hemolytic
disease, but subsequent pregnancies may
be at risk
c) The fetus is AT RISK for hemolytic disease only if the
biological father is Rh-negative
d) The fetus is AT RISK for hemolytic disease only if the
biological father is Rh-positive
CASE REPORT
A 5-year-old African-American male presents with
behavior problems noted in the first 3 months of
kindergarten. The mother explains that the child
does not pay attention and often naps in class. He
averages 10 hours of sleep nightly and is heard
snoring frequently. The mother has a history of
attention-deficit disorder and takes atomoxetine
(Strattera). The boy’s examination is within normal
limits except for his being in the 25th percentile for
weight and having 3+ tonsillar enlargement.
QUESTION 62
The most reasonable plan at this point would include
which one of the following?
ANSWER
a) Polysomnography
b) An electroencephalogram
c) Methylphenidate
d) Atomoxetine
CASE REPORT
A 16-year-old male is brought to your office by his
mother for “stomachaches.” On review of
systems, he also complains of headaches, occasional
bedwetting, and trouble sleeping. His examination is
within normal limits. His mother says that he is often
in the nurse’s office at school, and doesn’t seem to
have any friends. After some questions from you, he
admits to being called names and teased at school.
QUESTION 63
Which one of the following would be most
appropriate?
ANSWER
a) Explain that these symptoms are a stress reaction
and will lessen with time
b) Explain that he must try to conform to be more
popular
c) Order a TSH level
d) Explore whether his school counselor has a process
to address this problem
QUESTION 64
You are evaluating a 68-year-old male with obstructive
urinary symptoms. Which one of the following
medications may lead to falsely depressed levels of
prostate-specific antigen (PSA)?
ANSWER
a) Tamsulosin
b) Doxazosin
c) Lycopene
d) Finasteride
QUESTION 65
The most appropriate initial treatment for scabies in
an 8-year-old male is?
ANSWER
a) trimethoprim/sulfamethoxazole orally for 10 days
b) 0.5% malathion lotion
c) 5% permethrin cream
d) 5% precipitated sulfur in petroleum
CASE REPORT
A 3-year-old female presents with urinary
frequency, dysuria, and fever to 39.0° C (102.2° F).
She denies nausea, vomiting, fever, and flank pain.
There is no prior history of urinary infection and no
family history of urinary tract abnormalities. Urethral
catheterization reveals bacteriuria and a urine
culture reveals >100,000 colony-forming units of
Escherichia coli. She is started on appropriate
antibiotic therapy.
QUESTION 66
Evaluation to rule out anatomic abnormalities should
include?
ANSWER
a) renal ultrasonography and voiding
cystourethrography (VCUG) only if she has recurrent
infections
b) renal ultrasonography only if she has recurrent
infections
c) renal ultrasonography for this primary episode of
infection
d) renal ultrasonography and VCUG for this primary
episode of infection
CASE REPORT
A slender 22-year-old female is concerned about a
recent weight loss of 10 lb, frequent mild abdominal
pain, and significant diarrhea of 2 months’ duration.
Her physical examination is unremarkable, and
laboratory studies reveal only a moderate
microcytic, hypochromic anemia. Based on this
presentation.
QUESTION 67
which one of the following is the most likely diagnosis?
ANSWER
a) Villous adenoma
b) Ulcerative colitis
c) Infectious colitis
d) Celiac disease
QUESTION 68
A 73-year-old white male has severe COPD manifested
by repeated hospital admissions, 30-step dyspnea on
exertion, asthenia, and a resting pO of 58 mm Hg. Of
the following, which intervention will most likely
positively affect his survival?
ANSWER
a) Inhaled corticosteroids
b) Beta -Agonists
c) Oxygen supplementation
d) Pulsed antibiotic therapy
QUESTION 69
In patients with bipolar depression, monotherapy with
which one of the following may trigger a
manic episode?
ANSWER
a) Lithium
b) Carbamazepine
c) Lamotrigine
d) Fluoxetine
CASE REPORT
A 30-year-old female presents with concerns about
vaginal bleeding. She states that her menstrual
periods have occurred at regular intervals of 28–30
days for the past 15 years, but recently bleeding has
also occurred for a day or two in the middle of her
cycle. This bleeding has been heavy enough to
require the use of multiple pads.
QUESTION 70
Which one of the following terms best describes her
bleeding pattern?
ANSWER
a) Polymenorrhea
b) Metrorrhagia
c) Menometrorrhagia
d) Acute emergent abnormal uterine bleeding
QUESTION 71
A 25-year-old female presents with abdominal pain
localized to the right lower quadrant. Which one of
the following would be most helpful in diagnosing
acute appendicitis?
ANSWER
a) Abdominal/pelvic ultrasonography
b) A CBC
c) Plain abdominal films
d) Abdominal/pelvic C
QUESTION 72
A 17-year-old white female presents with new-onset
left-sided lower abdominal pain. Color flow Doppler
ultrasonography, in addition to pelvic
ultrasonography, would be most useful for
evaluating?
ANSWER
a) pelvic inflammatory disease
b) pelvic abscess
c) adnexal torsion
d) ruptured ovarian cyst
QUESTION 73
A 30-year-old female presents with a vaginal discharge. On
examination the discharge is homogeneous with a pH of
5.5, a positive whiff test, and many clue cells. Which one
of the following findings in this patient is most sensitive
for the diagnosis of bacterial vaginosis?
ANSWER
a) The character of the discharge
b) The presence of clue cells
c) The pH of the discharge
d) The whiff test
CASE REPORT
A 25-year-old female at 36 weeks gestation presents for
a routine prenatal visit. Her blood pressure is 118/78
mm Hg and her urine has no signs of protein or
glucose. Her fundal height shows appropriate fetal
size and she says that she feels well. On palpation of
her legs, you note 2+ pitting edema bilaterally.
QUESTION 74
Which one of the following is true regarding this
patient’s condition?
ANSWER
a) Her leg swelling requires no further evaluation
b) She most likely has deep venous thrombosis
c) She most likely has preeclampsia
d) You should order a 24-hr urine for protein
QUESTION 75
Which one of the following is most likely to induce
withdrawal symptoms if discontinued abruptly?
ANSWER
a) Divalproex
b) Venlafaxine
c) Fluoxetine
d) Olanzapine
CASE REPORT
A 78-year-old male comes to your office with a 3-day
history of pain in the right side of his chest. The pain
is described as burning and intense. Two days ago he
noted a rash at that site. Examination reveals groups
of vesicles on an erythematous base in a T-5
dermatome distribution on the right.
QUESTION 76
Which one of the following would be the most
appropriate treatment to minimize the chance of
post-herpetic neuralgia?
ANSWER
a) Carbamazepine
b) Capsaicin
c) Prednisone
d) Famciclovir
QUESTION 77
Which one of the following is the most common cause
of bacterial diarrhea?
ANSWER
a) Shigella dysenteriae
b) Escherichia coli O157:H7
c) Campylobacter jejuni
d) Salmonella enterica
CASE REPORT
A 28-year-old gravida 2 para 1 at 32 weeks gestation
presents with severe itching. She denies fever or
vomiting. Her physical examination is remarkable
for jaundice, but is otherwise benign. Laboratory
studies reveal a normal CBC, normal
platelets, normal glucose and serum creatinine
levels, normal transaminase levels, and a bilirubin
level of 4.0 mg/dL.
QUESTION 78
Which one of the following is the most likely diagnosis?
ANSWER
a) Serum hCG levels should double every 2–3 days if
the pregnancy is viable
b) Painless bleeding excludes the diagnosis of ectopic
pregnancy
c) Laparoscopy should be performed to exclude
ectopic pregnancy
d) A serum progesterone level >25 ng/mL indicates
that ectopic pregnancy is likely
CASE REPORT
A 35-year-old white female presents with a 6-month
history of irregular menstrual bleeding. Before this
problem began, her periods occurred every 30 days
and lasted 5 days. Now they occur every 20 days and
last for 10 days, and are heavier than they were
previously. A physical examination reveals no obvious
anatomic source of bleeding, and a Papanicolaou
(Pap) test is normal. A pregnancy test is negative and
a blood workup for organic causes of irregular
menses is also negative. She takes no medications.
QUESTION 79
Which one of the following would be most appropriate
at this point?
ANSWER
a) Reassurance that the problem will resolve on its
own
b) Changing to a progesterone-only contraceptive
c) Increasing the dosage of the oral contraceptive
d) Pelvic ultrasonography
QUESTION 80
Which one of the following is true regarding the risk of
physical spouse abuse?
ANSWER
a) It increases with alcohol and substance abuse
b) It decreases when a woman exits an abusive
relationship
c) It is higher among patients from racial minorities
d) It increases as socioeconomic status rises
QUESTION 81
Which one of the following causes rhinitis
medicamentosa with prolonged use in the treatment
of rhinitis?
ANSWER
a) Intranasal decongestants
b) Intranasal antihistamines
c) Intranasal mast cell stabilizers
d) Leukotriene antagonists
CASE REPORT
An 83-year-old female presents to your office as a new
patient. She recently moved to the area to be closer
to her family. A history reveals that she has been in
excellent health, has no complaints, and is on no
medications except occasional acetaminophen for
knee pain. She has never been in the hospital and
has not had any operations. She says that she feels
well. The examination is normal, with expected age-
related changes, except that her blood pressure on
three different readings averages 175/70 mm Hg.
These readings are confirmed on a subsequent
follow-up visit. In addition to lifestyle changes.
QUESTION 82
which one of the following would be most appropriate
for the initial management of this patient’s
hypertension?
ANSWER
a) A thiazide diuretic
b) An angiotensin receptor blocker
c) An ACE inhibitor
d) A beta-blocker
QUESTION 83
Surgery for obesity would be most appropriate for which one of
the following?
ANSWER
a) A 37-year-old African-American female who has failed
multiple diets, has no coexisting medical problems, and has a
BMI of 38
b) A 43-year-old African-American male with type 2 diabetes
mellitus who has failed multiple diets and has a BMI of 38
c) A 31-year-old white male who has lost 50 lb on his diet and
improved his sleep apnea, hypertension, and diabetes
mellitus, but still has a BMI of 42
d) A 45-year-old white female with a history of depression and
alcohol abuse who has failed multiple diets and has a BMI of
40
CASE REPORT
A 22-year-old gravida 2 para 1 presents to your office
with a 1-day history of vaginal bleeding and
abdominal pain. Her last menstrual period was 10
weeks ago, and she had a positive home pregnancy
test 6 weeks ago. She denies any passage of clots. On
pelvic examination, you note blood in the vaginal
vault. The internal cervical os is open.
QUESTION 84
Which one of the following best describes the patient’s
current condition?
ANSWER
a) Completed abortion
b) Threatened abortion
c) Incomplete abortion
d) Inevitable abortion
CASE REPORT
A 34-year-old white female visits your office with a chief
complaint of pelvic pain that intensifies with her menstrual
period. She has a history of pain during intercourse, which
started in her mid-twenties and has gradually become
worse. She reports recently missing some work during her
menstrual period due to the pain. She has had two
uneventful deliveries and the pain was absent during and
after each pregnancy, but gradually returned. She and her
husband do not wish to have any more children and her
husband has had a vasectomy. The patient denies vaginal
discharge or fever and a review of systems is negative. A
complete physical examination is normal except for
moderate nonspecific tenderness on pelvic examination. In
addition, her uterus is moderately retroverted and has
decreased mobility.
QUESTION 85
Which one of the following would be the most
appropriate initial step in the management of this
patient?
ANSWER
a) A complete hysterectomy and bilateral
oophorectomy
b) Conjugated estrogens
c) Depot medroxyprogesterone acetate
d) contraceptives
CASE REPORT
A 62-year-old male presents for surgical clearance prior
to transurethral resection of the prostate. His past
history is significant for a pulmonary embolus after a
cholecystectomy 15 years ago. His examination is
unremarkable except that he is 23 kg (50 lb)
overweight.
QUESTION 86
The most appropriate recommendation to the urologist
would be to?
ANSWER
a) place the patient on 650 mg of aspirin daily prior to
surgery
b) start warfarin after surgery with a goal INR of 1.5
c) start the patient on subcutaneous enoxaparin, 40
mg 1–2 hr prior to surgery and once a day after
surgery
d) cancel the surgery indefinitely
CASE REPORT
A 28-year-old white female sees you for preconception
counseling. For the past 3 years she has been
successfully treated with fluoxetine (Prozac) for
depression, and she asks if she can continue taking it
when she becomes pregnant. It is labeled by the FDA
as category C for use in pregnancy.
QUESTION 87
Which one of the following would you advise?
ANSWER
a) Animal studies do not indicate any risk to the fetus;
there are no studies in women
b) Animal studies demonstrate some risk to the fetus;
there are no studies in women
c) Controlled studies in women fail to demonstrate risk to
the fetus; it is safe to continue it
d) There is evidence of harm to the human fetus, and she
should discontinue it
QUESTION 88
Which one of the following is most predictive of
increased perioperative cardiovascular events
associated with noncardiac surgery in the elderly?
ANSWER
a) An age of 80 years
b) Renal insufficiency (creatinine 2.0 mg/dL)
c) Left bundle-branch block
d) A history of previous stroke
QUESTION 89
Which one of the following statements regarding
varicoceles is true?
ANSWER
a) The incidence of varicoceles in adult males is <5%
b) Varicoceles usually begin between 5 and 8 years of
age
c) A unilateral varicocele on the right side should be
referred for further evaluation
d) Most varicoceles are bilateral
QUESTION 90
A 2-year-old white female is brought to your office by
her parents, who are concerned about the child’s
“flat feet.” On evaluation, the child’s feet are flat with
weight-bearing, but with toe standing and with
sitting the arch appears. You would?
ANSWER
a) reassure the parents
b) recommend surgery
c) recommend foot-stretching exercises
d) recommend orthotics
QUESTION 91
When a woman less than 50 years of age develops
vulvar cancer, which one of the following associated
conditions is most frequently present?
ANSWER
a) Lichen sclerosus
b) Syphilis
c) Human papillomavirus
d) Lymphogranuloma venereum
QUESTION 92
A 35-year-old white male with known long QT
syndrome has a brief episode of syncope requiring
cardiopulmonary resuscitation. Which one of the
following is most likely responsible for this episode?
ANSWER
a) Asystole
b) Atrial flutter with third degree block
c) Sinus tachycardia
d) Torsades de pointes
QUESTION 93
Which one of the following is associated with a reduced
risk of postmenopausal osteoporosis?
ANSWER
a) Diuretic use
b) Cigarette smoking
c) Low BMI
d) Corticosteroid use
QUESTION 94
A 59-year-old female with type 2 diabetes develops a
2×1-cm ulcer on the plantar aspect of her right foot.
The ulcer is very deep and there is surrounding
cellulitis. A plain film is normal. Which one of the
following would be the imaging study of choice to
rule out osteomyelitis in this patient?
ANSWER
a) A leukocyte scan
b) A CT scan
c) Angiography
d) An MRI scan
CASE REPORT
• An 83-year-old female is admitted to the hospital
with an exacerbation of her COPD. On the second
hospital day she is clinically improved but is quite
disoriented, experiencing visual
hallucinations, agitation, and problems with recent
memory and attention span. She is noted bynthe
nursing staff to periodically fall asleep during
conversation. Her previous medical history is notable
for emphysema and hypertension, but there is no
history of psychiatric problems. Her blood pressure is
140/82 mm Hg, pulse 88 beats/min, and oxygen
saturation 98% on 2 L of nasal Oxygen.
QUESTION 95
Which one of the following does this patient most
likely have?
ANSWER
a) Schizophrenia
b) Mania
c) Acute depression
d) Delirium
QUESTION 96
A 31-year-old white female presents with her third
stress fracture of a lower extremity in the past4
years. Her history and examination are otherwise
unremarkable except for a controlled seizure
disorder. The most likely cause of her bone problem
is?
ANSWER
a) hypothyroidism
b) Addison’s disease
c) anticonvulsive medication
d) osteogenesis imperfecta
QUESTION 97
A 25-year-old female has unprotected intercourse and
chooses to use Plan B (two 0.75 mg tablets of
levonorgestrel, taken 12 hours apart) as a form of
emergency contraception. Plan B has been shown to
ANSWER
a) prevent implantation, ovulation, and fertilization
b) protect against sexually transmitted diseases
c) be teratogenic to an already established pregnancy
d) be effective only if used within 24 hours of
unprotected intercourse
QUESTION 98
The antimalarial agent mefloquine would be
contraindicated in which one of the following
patients?
ANSWER
a) A 50-year-old female with chronic renal failure and
a creatinine clearance of 50 mg/min
b) A 25-year-old female at 32 weeks gestation
c) A 55-year-old male with COPD who uses inhaled
albuterol and ipratropium bromide
d) A 45-year-old male with epilepsy controlled with
phenytoin.
CASE REPORT
Recent studies and meta-analyses have shown that
supplementation with a particular vitamin may do
more harm than good. This has resulted in a reversal
in the recommendation for its use in patients with
known coronary artery disease, and a
recommendation for caution in its use in general for
elderly patients.
QUESTION 99
This is true of which one of the following?
ANSWER
a) Vitamin A
b) Vitamin C
c) Vitamin B 12
d) Vitamin E
CASE REPORT
In early February, you receive a call from your office
nurse. Her 5-month-old daughter has been ill for
several days. What started as a mild upper
respiratory infection has progressed and she now has
profuse rhinorrhea, a temperature of 100.2° F (37.9°
C), and audible wheezing. In spite of an almost
nonstop cough, she does not appear acutely ill.
QUESTION 100
The organism responsible for this child’s illness is
most likely to be?
ANSWER
a) Bordetella pertussis
b) respiratory syncytial virus
c) group B Streptococcus
d) parainfluenza virus 3
QUESTION 101
Of the following, which is the most frequent cause of
seizures in the elderly?
ANSWER
a) Stroke
b) Head trauma
c) Dementia
d) Alcohol withdrawal
CASE REPORT
A 4-week-old full-term male is brought to your office by
his parents. They report that their child started
vomiting just after his 1-week visit. The parents are
concerned because they think the vomiting is
worsening, occurring after every feeding, and
“shooting across the room.” You note that the baby is
afebrile, but has not gained any weight since birth.
QUESTION 102
A 32-year-old gravida 3 para 2 is in labor at term following
an uncomplicated prenatal course. As you deliver the
fetal head it retracts against the perineum. Downward
traction fails to free the anterior shoulder. The most
appropriate course of action would be to?
ANSWER
a) have an assistant apply fundal pressure
b) place the mother’s thighs on her abdomen
c) apply increasingly strong downward traction to the fetal
head
d) deliberately fracture the clavicle of the fetus
CASE REPORT
A 14-year-old male is brought to your office by his
mother to establish care. The patient has been
diagnosed with asthma, but has not been on any
medications for the past year. When questioned, he
reports that his asthmatic symptoms occur daily and
more than one night per week. On examination, he is
found to have a peak expiratory flow of 75%.
QUESTION 103
Based on these findings, the most accurate
classification of this patient’s asthma is?
ANSWER
a) mild persistent
b) moderate persistent
c) severe persistent
d) mild intermittent
QUESTION 104
Which one of the following Papanicolaou (Pap) test
results is most likely to indicate a cancerous lesion?
ANSWER
a) High-grade squamous intraepithelial lesion (HSIL)
b) Atypical glandular cells not otherwise specified
(AGC-NOS)
c) Low-grade squamous intraepithelial lesion (LSIL)
d) Atypical squamous cells cannot exclude high-grade
intraepithelial lesion (ASC-H)
CASE REPORT
A 72-year-old female has stable but moderately
severe COPD requiring 2 L of continuous oxygen.
She plans to attend the college graduation of her
first grandchild, and wants to fly to avoid a 12-hour
car ride. Her PaO on room air is 55 mm Hg.
QUESTION 105
• According to the Federal Air Regulations, she MUST?
ANSWER
a) find a form of transportation other than air travel
b) bring a medical certificate from you certifying that
she is cleared to fly without oxygen
c) arrange through the airline for oxygen to be
available on board and in the airports
d) undergo preflight testing, including pulmonary
function testing and high-altitude simulation
testing, before she can be cleared to fly
QUESTION 106
Which one of the following criteria is most likely to signify
that an individual has bulimia nervosa rather than
anorexia nervosa?
ANSWER
a) The absence of at least three consecutive menstrual
cycles in a postmenarchal nonpregnant female
b) Recurrent episodes of binge eating
c) Intense fear of weight gain or becoming fat, even
though underweight
d) Denial of the seriousness of the current low body
weight
QUESTION 107
Painful ingrown toenails that display granulation tissue
and lateral nail fold hypertrophy are best
treated by?
ANSWER
a) removal of the entire nail
b) cotton-wick elevation of the affected nail corner
c) antibiotic therapy
d) excision of the lateral nail plate combined with
lateral matricectomy
CASE REPORT
A 36-year-old female makes an appointment because
her husband of 12 years was just diagnosed with
hepatitis C when he tried to become a blood donor
for the first time. He recalls multiple blood
transfusions following a motorcycle crash in 1988.
His wife denies past liver disease, blood
transfusions, and intravenous drug use. She has had
no other sexual partners. The couple has three
children.
QUESTION 108
Which one of the following is the best advice about
testing the wife and their three children?
ANSWER
a) No testing is required if her husband has normal
liver enzyme levels
b) No testing is required because tests have low
sensitivity
c) All family members should be tested because of
possible household fecal-oral spread
d) She should be offered testing because sexual
transmission is possible
QUESTION 109
You are evaluating a 5-month-old with
fever, tachypnea, and mild respiratory distress in the
emergency department. You hear mild basilar rales.
The child does not appear toxic. Which one of the
following tests would be the most appropriate as an
initial study?
ANSWER
a) A C-reactive protein level
b) A chest radiograph
c) A CBC
d) Oxygen saturation by pulse oximetry
QUESTION 110
The elderly patient may process medications differently
than a younger adult. Which one of the following is
LEAST affected by aging?
ANSWER
a) Absorption
b) Distribution
c) Compliance
d) Metabolism
QUESTION 111
A 72-year-old Asian female is found to have asymptomatic
gallstones on abdominal ultrasonography performed to
evaluate an abdominal aortic aneurysm. Which one of
the following would be the most appropriate
management for the gallstones?
ANSWER
a) Lithotripsy
b) Laparoscopic cholecystectomy
c) Open cholecystectomy
d) Observation
• A 34-year-old female presents to you for
preconception counseling regarding the
management of her chronic hypertension. Her
blood pressure has been well controlled on
benazepril, 20 mg/day, without any side effects.
The patient’s blood pressure was 145/95 mm Hg
prior to beginning benazepril. She has been
pregnant once before, and her physician switched
her to methyldopa during that pregnancy, but she
suffered from drowsiness and a dry mouth during
much of that time. The pregnancy and delivery
were otherwise uncomplicated. She has no
history of diabetes mellitus, renal insufficiency, or
asthma. She is a nonsmoker.
QUESTION 112
• Which one of the following would you do when she
becomes pregnant?
ANSWER
a) Switch to atenolol until after delivery
b) Switch to long-acting nifedipine until after delivery
c) Continue the benazepril through the pregnancy and
delivery
d) Discontinue the benazepril and monitor closely
throughout the pregnancy for signs of
preeclampsia or fetal growth restriction
QUESTION 113
Which one of the following is the preferred treatment
for patients with obsessive-compulsive disorder?
ANSWER
a) Valproic acid
b) Alprazolam
c) Fluoxetine
d) Lithium carbonate
QUESTION 114
When repairing a perineal laceration after a vaginal
delivery, which one of the following suture materials
decreases both wound dehiscence and postpartum
perineal pain?
ANSWER
a) Polyglactin 910 (Vicryl)
b) Silk
c) Plain catgut
d) Polypropylene
QUESTION 115
A 40-year-old female is scheduled for a
cholecystectomy and you wish to estimate her risk
for postoperative bleeding. Which one of the
following provides the most sensitive method for
identifying her risk?
ANSWER
a) Prothrombin time (PT)
b) Activated partial thromboplastin time (aPTT)
c) Bleeding history
d) Bleeding time
QUESTION 116
A mother is nearing the end of maternity leave. She
asks for your advice regarding breastfeeding her
infant after she returns to work. Which one of the
following would be accurate advice?
ANSWER
a) Breast milk should be stored in glass bottles
b) Separation of breast milk indicates spoilage
c) Breast milk should be thawed in boiling water
d) Refreezing breast milk destroys some proteins
QUESTION 117
Compared to anesthesia using only parenteral
opioids, the use of epidural anesthesia in labor and
delivery increases the rate of which one of the
following?
ANSWER
a) Low Apgar scores (<7)
b) Maternal low backache 3 months post delivery
c) Prolonged second stage of labor
d) Cesarean section
CASE REPORT
A 70-year-old white female presents with a pruritic rash
of her sacrum that has occurred intermittently over
the last 6 years. She reports that the area is always
very tender just before the blister-like lesions erupt.
She is otherwise in good health, and takes no
medications. Her past medical history is
unremarkable. You provide appropriate treatment
for the condition.
QUESTION 118
You should advise the patient to avoid which one of
the following during future outbreaks?
ANSWER
a) Sexual contact
b) Prolonged sitting
c) Sun exposure
d) Excessive intake of green, leafy vegetables
• A 32-year-old female presents with bilateral
pretibial tender, mildly red nodules 2–4 cm in
diameter. A nodule that appeared earlier
resolved, leaving a “bruised” area. She had a
similar problem once when she was pregnant but
it resolved spontaneously. Her medications
include lovastatin (Mevacor) for hyperlipidemia
and a low-dose oral contraceptive prescribed 5
months earlier. Her past history and a review of
systems are otherwise unremarkable.
QUESTION 119
The most appropriate next step would be to
ANSWER
a) order a serum creatine phosphokinase level
b) discontinue her oral contraceptive
c) obtain a cervical culture for gonorrhea
d) discontinue lovastatin
QUESTION 120
A 23-year-old female was recently diagnosed with
bipolar disorder after experiencing her first episode
of acute severe mania. Which one of the following
would be appropriate initial maintenance therapy for
her?
ANSWER
a) No medication unless she has a second severe
episode
b) Valproate
c) Gabapentin
d) Lorazepam
CASE REPORT
A 23-year-old sexually active female presents to your
office with a 2-week history of vaginal discharge and
mild coital discomfort. On physical examination, you
note the presence of a mucopurulent vaginal
discharge and cervical friability. She is afebrile and
there are no other positive physical findings. No
trichomonads or yeast is seen on vaginal
preparations. Material for Chlamydia trachomatis–
specific DNA testing is submitted and results will be
available in 2 days.
QUESTION 121
Which one of the following is true regarding appropriate
management?
ANSWER
a) No treatment is indicated until laboratory results are
known
b) metronidazole given now will clear her discharge
c) A 7-day course of doxycycline is superior to a 1-g dose
of azithromycin
d) The patient should be instructed to refrain from sexual
intercourse until 7 days after
initiating therapy
QUESTION 122
Compared to children with ADHD, adults with ADHD?
ANSWER
a) tend to be less impulsive
b) tend to be more hyperactive
c) are less likely to complain of inattention difficulties
d) are less likely to have corroboration of symptoms by
family members
QUESTION 123
A 33-year-old female requests combined oral
contraceptive pills (OCPs) for birth control. Which
one of the following would be a contraindication to
prescribing OCPs for this patient?
ANSWER
a) A family history of ovarian cancer
b) A history of controlled hypertension
c) A history of hepatitis C infection with no liver
disease
d) A history of thromboembolic disease
QUESTION 124
Routine blood tests frequently reveal elevated calcium
levels. When this elevation is associated with
elevated parathyroid hormone levels, which one of
the following is an indication for parathyroid surgery?
ANSWER
a) Concurrent hyperthyroidism
b) Increased bone density
c) Age >50
d) Kidney stones
QUESTION 125
A 21-year-old female had a pelvic examination and a
normal Papanicolaou (Pap) test 3 months ago. She is now
being treated for chlamydial cervicitis with
azithromycin, 1 g in a single dose. When should she next
have a test for Chlamydia?
ANSWER
a) Never, unless she is symptomatic or has a suspected
exposure to Chlamydia at some
point in the future
b) 3–4 months
c) 1–2 weeks
d) 9 months (at her next routine pelvic examination)
QUESTION 126
A 10-year-old male develops a fever 3 months after
returning to the U.S. following a year in the tropics. He
was treated with the proper immunizations and given
appropriate prophylactic medications before he went
abroad with his parents. Which one of the following
tropical illnesses should be considered in the differential
diagnosis?
ANSWER
a) Typhoid fever
b) Dengue fever
c) Plague
d) Malaria
CASE REPORT
A mother brings her 12-month-old son to your
clinic, concerned that he is repeatedly banging his head
against the floor, wall, or crib. She reports that this
behavior began about 2 months ago. It now occurs
several times per week, and at times is incited when
the child is frustrated with a toy or when he does not
get what he wants from his parents. The mother notes
that she is sometimes awakened at night by the sound
of her son rhythmically banging his head against the
rail of hiscrib. Physical examination reveals a normal
child with some soft-tissue swelling of the
forehead, but no broken skin, ecchymosis, or signs of
bony damage. Developmental milestones and growth
have been normal, and the child is not on any
medications.
QUESTION 127
Children with this presentation are most likely to have
which one of the following?
ANSWER
a) A skin laceration or skull fracture
b) An eventual diagnosis of Lesch-Nyhan syndrome
c) Extinction of this habit by age 3
d) Future cognitive delay when compared with
children without this habit
CASE REPORT
A 4-year-old white female is brought to your office by
her mother, who reports that the child recently
developed a foul-smelling vaginal discharge. After an
appropriate history and general examination, you
determine that a genital examination is necessary.
QUESTION 128
Which one of the following positions is most likely to
allow for visualization of the child’s vagina and cervix
without instrumentation?
ANSWER
a) The knee-chest position on an examination table
b) Trendelenburg’s position on an examination table
c) The left lateral decubitus position on an
examination table
d) Supine in the mother’s lap
QUESTION 129
Which one of the following has been shown to transmit
Salmonella infections to humans?
ANSWER
a) Iguanas
b) Guinea pigs
c) Civet cats
d) Rabbits
QUESTION 130
The likelihood of postpartum depression is increased by
which one of the following?
ANSWER
a) A prior history of depression
b) Bottle feeding
c) Low educational level
d) Cesarean delivery
QUESTION
A 31-year-old female presents with her first outbreak of
genital herpes. She has been married for 4 years and
says she has not had sexual relations with anyone
other than her husband since they started dating.
QUESTION 131
When counseling this patient, which one of the following
would be appropriate advice?
ANSWER
a) Suppressive therapy can decrease transmission to
sexual partners
b) Daily application of topical acyclovir is effective for
suppressant therapy
c) If type-specific serologies are negative, genital herpes
can be ruled out
d) Because the average incubation period for herpes is 3
months, her husband has most likely had another
sexual partner in the past few months
CASE REPORT
A 32-year-old female is concerned about ovarian
cancer. She has no symptoms at this time.
However, she has a close friend who was recently
diagnosed with ovarian cancer at an advanced
stage. This friend told the patient that a “simple
blood test” for CA-125 could detect ovarian
cancer at a curable stage.
QUESTION 132
As part of your discussion, you inform the patient that
which one of the following is true?
ANSWER
a) A high serum CA-125 level is not associated with
ovarian cancer
b) Most consensus opinions recommend performing
this test for average-risk women
c) This test should not be ordered due to its high false-
negative rate
d) Detecting ovarian cancer at an earlier stage using
serum CA-125 has not been shown to reduce
mortality
QUESTION 133
Ehrlichiosis may result from exposure to?
ANSWER
a) freshwater snails
b) fleas
c) ticks
d) rats
QUESTION 134
The best method of influenza prophylaxis in high-risk
patients is administration of?
ANSWER
a) oral amantadine daily during the influenza season
b) influenza vaccine intramuscularly prior to the
influenza season
c) oral rimantadine daily during the influenza season
d) oral oseltamivir daily during the influenza season
QUESTION 135
A 75-year-old male is brought in by his family due to
concerns about memory loss. Which one of the
following is best for determining whether or not he
has dementia?
ANSWER
a) A Mini-Mental State Examination (MMSE)
b) Non–contrast-enhanced CT of the head
c) An electroencephalogram
d) MRI of the head
QUESTION 136
Of the following, which one has the greatest potential
to spread via human-to-human transmission?
ANSWER
a) Pneumonic plague
b) Botulism
c) Typhoidal tularemia
d) Coxiella burnetii (Q fever)
QUESTION 137
The mother of a 3-year-old male is concerned that he
doesn’t like being held, doesn’t interact much with
other children, and rarely smiles. Of the
following, which feature would be most helpful in
distinguishing Asperger’s syndrome from autism in
this patient?
ANSWER
a) Repetitive fine motor mannerisms
b) Delayed gross motor development
c) Normal language development
d) Preoccupation with parts of objects
CASE REPORT
A 76-year-old male is brought to your office by his son. The
patient complains of dizziness that has slowly been
worsening over the past year. His description is vague, but
he says that he notices the dizziness when he tries to walk.
The review of systems is normal, except for increasing
problems with his “prostate” in the last 6 months, with
dribbling and accidents at times. The patient admits to
“going more often,” and sometimes “without warning.” The
son states that his father seems more forgetful, slower of
speech, and not as “full of life” as he used to be. A
neurologic examination reveals the patient to be oriented ×
3, with a somewhat flat affect and a wide-
based, slow, shuffling gait. The examination is otherwise
normal. His Mini-Mental State Examination (MMSE) score is
22 out of 30. No tremor is noted. A CBC, creatinine
level, TSH level, vitamin B 12 level, and VDRL are all normal.
QUESTION 138
Which one of the following should you do next?
ANSWER
a) Order a brainstem auditory evoked potential
b) Prescribe sertraline (Zoloft)
c) Order physical therapy
d) Order a brain MRI
QUESTION 139
The most appropriate initial pharmacologic treatment
of panic disorder is?
ANSWER
a) valproic acid (Depakene)
b) lithium
c) an SSRI
d) a tricyclic antidepressant
CASE REPORT
A 24-year-old worker at a local day-care center has not
had a menstrual period in over 2 months. A home
pregnancy test is positive and is confirmed in your
office. She is concerned because she was recently
exposed to a child who was diagnosed with rubella.
A serum rubella IgG antibody test is negative.
QUESTION 140
Which one of the following would be the best course
of action?
ANSWER
a) Order level III ultrasonography to screen for
congenital anomalies
b) Repeat the serologic testing in 2–3 weeks
c) Administer rubella immune globulin
d) Tell the patient she is immune to rubella and is not
at risk
CASE REPORT
A 23-year-old male with Down syndrome is brought to
your office by his parents. The patient has had a low-
grade fever (approximately 100° F) for 7 days. The
fever is not associated with rhinorrhea, sore
throat, cough, dysuria, or shortness of breath. The
parents note that their son has complained of aches
in various places, and has not been eating well for
several weeks. The physical examination is
remarkable for swollen lymph glands in multiple
places, pale conjunctivae, and an 8-lb weight loss
since his last visit 1 month ago.
QUESTION 141
Which one of the following is most consistent with
this patient’s symptoms?
ANSWER
a) Urinary tract infection
b) Atypical pneumonia
c) Acute myeloid leukemia
d) Hodgkin’s lymphoma
QUESTION 142
Which one of the following is true regarding Osgood-
Schlatter disease?
ANSWER
a) Complete symptom resolution usually occurs in 18–24
months
b) Effusions are often present on examination and knee
range of motion is limited
c) Most cases are triggered by a single traumatic event
d) Pain is localized to the inferior patellar pole at the origin
of the patellar tendon
QUESTION 143
Which one of the following is an indicator of low risk
for streptococcal infection in a patient with acute
pharyngitis?
ANSWER
a) Soft palate petechiae
b) Cough
c) Anterior cervical lymphadenopathy
d) Fever
QUESTION 144
Your community recently experienced an outbreak of
infectious diarrheal illness due to the protozoan
Cyrptosporidium, a chlorine resistent organism. A
reporter from the local newspaper asks you if there are
other chlorine-resitant fecal organisms that could
contaminate public drinking water. You would tell the
reporter that such organisms include. 15/20 missed
ANSWER
a) Escherichia Coli
b) Giardia Lamblia
c) Rotavirus
d) Campylobacter jejuni
CASE REPORT
A 32-year-old white female comes to your office
complaining of dysuria. She denies fever , back
pain, and urinary frequency. She appears to be well
otherwise, and has a normal abdominal examination. A
clean-catch urinalysis shows 15-20 WBCs/hpf and a
dipstick test for leukocyte esterase is positive. You send
a urine sample for culture and start the patient on
nitrofurantoin (Macrodantin), as she is allergic to sulfa.
Three days later, the patient returns with persistent
dysuria despite taking the medication as prescribed.
Her urine culture has returned with no growth. A pelvic
examination is normal, and the rest of the physical
examination is unchanged. A wet prep is normal and
tests for sexually transmitted diseases are pending.
QUESTION 145
Which one of the following antibiotics is most
appropriate for this patient now?
ANSWER
a) Metronidazole
b) Doxycycline
c) Cephalexin
d) Ciprofloxacin
QUESTION 146
In which one of the following would misoprostol be
appropriate?
ANSWER
a) A primigravida at term whose cervix is dilated to 7 cm
and whose frequency of contractions has slowed
b) A primigravida with a long, thick cervix at term who has
a previous history of myomectomy
c) A diabetic primigravida with a long, thick cervix at 38
weeks gestation for whom induction is indicated
d) A gravida 2 para 1 with a long, thick cervix at term who
is interested in vaginal birth after a cesarean section
QUESTION 147
Which one of the following therapeutic interventions
is useful in the treatment of croup, but is not an
accepted treatment for bronchiolitis?
ANSWER
a) Oral dexamethasone
b) Extracorporeal membrane oxygenation
c) Nebulized racemic epinephrine
d) Oxygen via the blow-by method
CASE REPORT
Your office nurse reports an accidental needlestick with
a used venipuncture needle. She had drawn blood
from a 35-year-old white male, who is a new patient
undergoing evaluation for hypertension and
hyperlipidemia. He has no evidence of other health
problems and no risk factors for HIV. Both the nurse
and the patient are found to be HIV negative.
QUESTION 148
Which one of the following would be the most appropriate
advice?
ANSWER
a) No further testing for HIV is necessary
b) She should begin treatment with prophylactic
antiretroviral therapy within the next 48 hours
c) She should refrain from unprotected sexual contact
with her husband until another negative test 6 weeks
after the exposure
d) She should have repeat HIV testing at 6, 12, 24, and 52
weeks after the exposure
QUESTION 149
Which one of the following is consistent with current
recommendations?
ANSWER
a) Cervical cytologic screening should be done yearly prior to
the age of 30, with either a conventional Pap smear or
liquid-based cytology
b) Cervical cancer screening should begin no later than 18
years of age in all women
c) Cervical cancer screening with Papanicolaou (Pap) smears
should begin within 1 year after the onset of vaginal
intercourse
d) Screening with vaginal cytology is not indicated in women
who have had a total hysterectomy for benign gynecologic
disease
QUESTION 150
The predominant symptom associated with dysthymic
disorder is?
ANSWER
a) loosening of associations
b) delusions
c) incoherence
d) depression
QUESTION 151
Which one of the following is a common early side
effect of fluoxetine?
ANSWER
a) Skin rash
b) Orthostatic hypotension
c) Constipation
d) Loss of appetite
CASE REPORT
You see a 17 year old white female who has recently
become sexually active. She requests oral
contraceptives and you perform a brief
evaluation, including blood pressure measurment. A
Pregnancy test is negative. She is resistant to further
evaluation unless it is necessary.
QUESTION 152
In addition to appropriate counseling, which one of the
following should be done before prescribing oral
contraceptives? 17/20 missed.
ANSWER
a) No further evaluation at this visit unless indicated
by history.
b) A pelvic examination and Papanicolaou test
c) Screening for sexully transmitted diseases.
d) A breast exam.
CASE REPORT
You diagnose depression in a 27 year old whit male
and prescribe buproprion. He returns for a follow
up visit 4 weeks later. He now reports less
despondency, but says he has developed severe
insomnia. He also report that his mind "races" all
the time, and that he has recieved two speeding
tickets in the past 2 weeks
QUESTION 153
Which one of the following would be appropriate
now?
ANSWER
a) Add Valproate
b) Add imipramine at bedtime
c) Substitute sertraline for the buproprion
d) Add Alprazolam
CASE REPORT
A 73-year-old white female presents to the office with her
daughter. She complains of a several-year history of
intermittent involuntary loss of urine which is gradually
worsening. She frequently loses small amounts of urine
when she rises from a low chair, and greater amounts if she
coughs, sneezes, or laughs. She has taken to wearing
absorbent undergarments and admits to significant
embarrassment and limitation of activities as a result of her
problem. Her daughter states that the patient has gradually
withdrawn from her usual social contacts. Physical
examination reveals a well-appearing elderly female. Her
pelvic examination shows mucosal atrophy and a mild
cystocele. With a full bladder, she loses a few ml of urine
into a gauze pad when standing and coughing. Her post-
void residual is normal. Her physical examination is
otherwise normal for her age, and her urinalysis is normal.
QUESTION 154
If nonpharmacologic interventions are not
completely successful in the management of this
patient's problem, which one of the following
medications is most likely to help? 19/20 missed.
ANSWER
a) Bethanechol
b) Hycosamine
c) Flavoxate
d) Pseudoephedrine
QUESTION 155
Which one of the following is the medical treatment
of choice for acute delirium in the intensive care
unit?
ANSWER
a) Intravenous droperidol every 6-8 hours
b) Intravenous haloperidol in increasing doses
every 30 minutes as needed
c) Intramuscular diphenhydramine
d) Intramuscular chlorpromazine
QUESTION 156
Screening for Chlamydia trachomatis infection should be
performed on which one of the following asymptomatic
patients who present for routine evaluation?
ANSWER
a) A 20-year-old female who uses birth control pills for
contraception and has a new sexual partner
b) A 22-year-old male with multiple sexual partners who
uses condoms, but not consistently
c) A 16-year-old female who states that she is not sexually
active
d) A 28-year-old female who is married, has no history of
sexually transmitted disease, and uses the rhythm method
to avoid pregnancy
QUESTION 157
The most common hospital errors associated with
preventable adverse drug effects are in the stage
of 16/20 missed?
ANSWER
a) transcription
b) dispensing
c) administration
d) ordering
CASE REPORT
A 73-year-old white female complains of difficulty
reading for the past several months. Examination
reveals 20/100 near vision bilaterally, and normal
visual fields by confrontation. An ophthalmoscopic
examination reveals only small yellow lesions
clustered in the retina.
QUESTION 158
The most likely diagnosis is?
ANSWER
a) glaucoma
b) retinal detachment
c) cataract
d) macular degeneration
QUESTION 159
The most effective way to diagnose chronic alcoholism
is to?
ANSWER
a) ask the patient directly if he/she is an alcoholic
b) confront the patient when he/she is intoxicated
c) obtain a careful history of alcohol intake from the
patient
d) inquire about problems resulting from drinking
QUESTION 160
Compared with younger adults, healthy older adults?
ANSWER
a) spend fewer hours in stages of deep sleep
b) fall asleep more quickly at bedtime
c) spend less time awake in bed
d) awaken less frequently during the night
QUESTION 161
Which one of the following is more likely to occur
with glipizide than with metformin?
ANSWER
a) Lactic acidosis
b) Gastrointestinal distress
c) Hypoglycemia
d) Weightloss
QUESTION 162
While examining a male infant during a routine well child
visit, you palpate only one testicle within the scrotum.
Which one of the following is true concerning this
problem?
ANSWER
a) The risk of malignancy in the undescended testis is
several times higher than in the general population
b) Surgery should be postponed until 3 years of age to
allow for spontaneous descent
c) Orchiopexy with antecedent hormonal therapy (LH-
RH, hCG) is the currently recommended treatment
d) In spite of orchiopexy, more than 50% of patients with a
history of an undescended testis are infertile as adults
CASE REPORT
A 21-year-old white female presents to the emergency
department with a history consistent with a lateral
ankle sprain that occurred 2 hours ago while she was
playing softball. She complains of pain over the distal
anterior talofibular ligament, but is able to bear
weight. There is mild swelling, mild black and blue
discoloration, and moderate tenderness to palpation
over the insertion of the anterior talofibular
ligament, but the malleoli are nontender to
palpation.
QUESTION 163
Which one of the following statements is true regarding
the management of this case? 13/20 missed.
ANSWER
a) Anteroposterior, lateral, and 30° internal oblique
(mortise view) radiographs should be done to rule out
fracture
b) For best results, functional rehabilitation should begin
within the first 24 hours after injury
c) Stress radiographs will be needed to rule out a major
partial or complete ligamentous tear
d) Early range-of-motion exercises should be initiated to
maintain flexibility
QUESTION 164
In evaluating an adult with anemia, which one of the
following findings most reliably indicates a
diagnosis of iron deficiency anemia?
ANSWER
a) Microcytosis
b) Low serum ferritin
c) Low total iron-binding capacity
d) Hypochromia
QUESTION 165
During a routine well child examination of a 9-month-
old female, you discover labial adhesions with a 5-
mm opening anteriorly. Which one of the following
statements is true concerning this condition?
ANSWER
a) It is rarely associated with urinary tract infections
b) The labia majora are most likely affected
c) It is rarely found in neonates
d) Oral estrogens are an accepted alternate treatment
CASE REPORT
A 37-year-old gravida 6 para 5 is given oxytocin to
induce delivery at 41 weeks gestation. Her prenatal
course is significant for chronic hypertension. She
delivers a 4020-g (8 lb 14 oz) baby. Soon after
delivery of the placenta, she begins to have excessive
vaginal bleeding.
QUESTION 166
Which one of the following would you do initially?
ANSWER
a) Chromosome analysis
b) A TSH level
c) Phenylketonuria screening
d) A serum creatine kinase level
QUESTION 167
Of the following antidepressants, which one is LEAST
likely to cause drug interactions?
ANSWER
a) Citalopram
b) Paroxetine
c) Fluoxetine
d) Mirtazapine
QUESTION 168
When advising women about the need for breast cancer
screening, which one of the following statements is
accurate regarding risk factors and the need for screening?
ANSWER
a) Hereditary breast cancer is more common in
postmenopausal women
b) Having several second degree relatives with breast cancer
carries the same risk as having one first degree relative
with breast cancer
c) Most women with breast cancer have no identifiable risk
factors
d) When hereditary breast cancer strikes several generations
of a family, women descended from a male family
member are not at increased risk
QUESTION 169
Of the following, the INITIAL treatment of choice in
the management of severe hypertension during
pregnancy is?
ANSWER
a) labetalol intravenously
b) reserpine intramuscularly
c) nifedipine sublingually
d) enalapril intravenously
QUESTION 170
A 35-year-old female complains that her nose is too
large, even after having cosmetic surgery on her nose
three times. She has a minimal social life because of
her concern about the appearance of her nose. A
physical examination, including her appearance, is
normal. What is the most likely diagnosis?
ANSWER
a) Body dysmorphic disorder
b) Social phobia
c) Obsessive-compulsive disorder
d) Delusional disorder
QUESTION 171
A 39-year-old white male comes to your office with
severe pain, fluctuance, erythema, and, tenderness
localized over the pad of the distal long finger. The
most likely diagnosis is?
ANSWER
a) digital cellulitis
b) a felon (whitlow)
c) a paronychia
d) an eponychial abscess
CASE REPORT
You see a 24-year-old obstetric patient late in her
third trimester for mild dysuria and urinary
urgency. Microscopic examination of the urinary
sediment is notable for bacteria and you make a
presumptive diagnosis of cystitis.
QUESTION 172
Which one of the following antibiotics would be
CONTRAINDICATED?
ANSWER
a) Trimethoprim/sulfamethoxazole
b) Cephalexin
c) Nitrofurantoin
d) Amoxicillin-clavulanate
QUESTION 173
The antiphospholipid syndrome in women is
commonly associated with
ANSWER
a) recurrent abortion
b) secondary amenorrhea
c) metrorrhagia
d) dysmenorrhea
QUESTION 174
Which one of the following statements is true regarding
the association between oral contraceptive use and the
risk of breast cancer?
ANSWER
a) The longer the duration of oral contraceptive use, the
greater the risk of breast cancer
b) The risk of breast cancer is directly correlated with a
higher estrogen content in the oral contraceptive
c) The risk of breast cancer in women who use oral
contraceptives is linked to a family history of breast
cancer
d) There is no significant difference in the risk of breast
cancer between women who have used oral
contraception and those who have not
CASE REPORT
A 67-year-old white male sees you for a new
patient visit. He is asymptomatic and has not
seen a doctor in 10 years. He does not smoke or
drink and takes no medication. He says he has a
history of "mild high blood pressure" but has
never been treated for this. His blood pressure
today is 180/90 mm Hg. He has a decreased
arteriovenous ratio on funduscopic
examination, his point of maximal intensity is
displaced laterally, and he has decreased pedal
pulses.
QUESTION 175
The most appropriate management at this point would
be to?
ANSWER
a) perform two blood pressure measurements 1 week
apart to establish the diagnosis of hypertension
b) prescribe a diuretic
c) order a laboratory workup to rule out causes of
secondary hypertension
d) order ambulatory blood pressure monitoring
QUESTION 176
You see a 90-year-old male with a 5-year history of
progressive hearing loss. The most common type of
hearing loss at this age affects?
ANSWER
a) all frequencies roughly the same
b) predominantly high frequencies
c) predominantly low frequencies
d) predominantly mid-frequencies
QUESTION 177
A 24-year-old white female presents to the office with a
6-month history of abdominal pain. A physical
examination, including pelvic and rectal
examinations, is normal. Which one of the following
would indicate a need for further evaluation?
ANSWER
a) Changes in stool consistency from loose and watery
to constipation
b) Worsening of symptoms at night
c) Passage of mucus with bowel movements
d) Abdominal bloating
QUESTION 178
Which of the following is the most reliable clinical
symptom of uterine rupture? 14/20 missed.
ANSWER
a) Vaginal bleeding
b) Fetal distress
c) Sudden tearing uterine pain
d) Loss of uterine tone
QUESTION 179
Which one of the following is a risk factor for preterm
birth?
ANSWER
a) High prepregnancy weight
b) Prior preterm delivery
c) Smoking
d) First-trimester bleeding
QUESTION 180
The hospital nursery reports that a 24-hour-old male
has developed "acne" confined to his nose and
cheeks. Your examination confirms the presence of
acneiform lesions,including papules. Which one of
the following would be most appropriate?
ANSWER
a) Clindamycin topically
b) Boric acid rinses
c) No treatment
d) An emollient lotion
QUESTION 181
A 38-year-old alcoholic male has successfully
completed outpatient alcohol detoxification and has
plans to participate in Alcoholics Anonymous. Which
one of the following pharmacologic agents can aid in
relapse prevention?
ANSWER
a) Naloxone
b) Bupropion
c) Naltrexone
d) Mirtazapine
CASE REPORT
An 85-year-old white male is brought to you
for the first time by his son. The father has
recently seen a neurologist who performed
a workup for dementia and diagnosed
moderate Alzheimer's disease.
QUESTION 182
Which one of the following is true regarding the use
of a cholinesterase inhibitor in this patient?
ANSWER
a) Agitation is often intensified by these agents
b) If the patient has a vascular dementia rather than
Alzheimer's dementia the drug will not be useful
c) Nursing-home placement may be delayed a year
or longer
d) Memory is likely to improve significantly
QUESTION 183
In a 27-year-old white female with irregular
menstrual cycles and infertility , which one of the
following would be more indicative of Cushing , s
syndrome rather than the more common
polycystic ovarian syndrome?
ANSWER
a) Acne
b) Hirsutism
c) Acanthosis nigricans
d) Easy bruising
QUESTION 184
You diagnose attention-deficit/hyperactivity disorder in a 10-
year-old male, and the parents ask about drug
treatment. Which one of the following would you tell the
parents with regard to potential side effects of drug
treatment?
ANSWER
a) The effect on eventual adult height is minimal in patients
treated with methylphenidate or dextroamphetamine
b) Methylphenidate has significantly fewer side effects than
dextroamphetanrine
c) Although anorexia may occur, it will not lead to substantial
weight loss
d) Methylphenidate may precipitate seizures
CASE REPORT
A 60-year-old female has been on conjugated equine
estrogens/medroxyprogesterone (Prempro) since she
went through menopause at age 52. She still has her
uterus and ovaries. She is having no side effects that
she is aware of and is experiencing no vaginal
bleeding. She is worried about the health effects of
her hormone replacement therapy and asks your
advice about risks versus benefits.
QUESTION 185
Which one of the following would be accurate advice
regarding these risks and benefits?
ANSWER
a) The incidence of myocardial infarction is decreased
b) The incidence of pulmonary embolus is decreased
c) The incidence of colorectal cancer is increased
d) The incidence of breast cancer is increased
QUESTION 186
Which on of the following is the most effective initial
treatment of head lice in an 8 year old child?
ANSWER
a) Head Shaving
b) Malathion
c) Wet combing every 4 days, to continue for 2 weks
after any louse is found
d) Lindane
QUESTION 187
Which one of the following is associated with an
increased risk of developing Alzheimer's disease?
ANSWER
a) A low-fiber diet
b) A positive homozygous genotype for
apolipoprotein E4
c) Elevated serum aluminum
d) Decreased serum B-carotene
QUESTION 188
Which one of the following is true regarding testicular
torsion?
ANSWER
a) It is most common in boys under the age of 10
b) Testicular scanning is necessary prior to surgical
exploration in order to document loss of blood flow
to the testis
c) Testicular torsion is often preceded by a
communicating hydrocele
d) Surgical exploration and detorsion must be done
within 6 hours of the onset in order to preserve
testicular function
QUESTION 189
Which one of the following can contribute to serum
calcium elevation?
ANSWER
a) Hydrochlorothiazide
b) Enalapril
c) Verapamil
d) Allopurinol
QUESTION 190
Which one of the following intravenous antibiotic
regimens is most appropriate for the treatment of
postpartum endometritis?
ANSWER
a) Ciprofloxacin plus doxycycline.
b) Gentamicin plus methicillin
c) Ciprofloxacin plus nafcillin
d) Gentamicin plus clindamycin
QUESTION 191
Female victims of domestic assault?
ANSWER
a) seldom spontaneously offer accounts of partner
abuse
b) usually seek medical care for their injuries
c) are less likely to be abused during pregnancy
d) usually resent unsolicited questions about abuse
CASE REPORT
To establish a reference range for a particular blood
test in pregnant teenagers, your laboratory has
plotted measurements for a large cohort of healthy
subjects. The laboratory finds that these
measurements fit a Gaussian distribution, and sets
its reference range to include measurements within
two standard deviations above and below the
mean.
QUESTION 192
What is the approximate probability that a
healthy person will have a test result that falls
outside the laboratory reference range?
ANSWER
a) 10%
b) 0.01%
c) 0.1%
d) 5%
QUESTION 193
If an abnormally high maternal serum alpha
fetoprotein level is found at 16-18 weeks
gestation, the next step is?
ANSWER
a) repeating the measurement of the maternal serum
alpha fetoprotein at 24 weeks gestation.
b) amniocentesis for amniotic fluid alpha
fetoprotein, acetylcholinesterace, and karyotyping.
c) recommending termination of the pregnancy.
d) high resolution ultrasonography
Segundo simulador primera parte
Segundo simulador primera parte
Segundo simulador primera parte
Segundo simulador primera parte
Segundo simulador primera parte
Segundo simulador primera parte
Segundo simulador primera parte
Segundo simulador primera parte
Segundo simulador primera parte
Segundo simulador primera parte
Segundo simulador primera parte
Segundo simulador primera parte
Segundo simulador primera parte
Segundo simulador primera parte
Segundo simulador primera parte
Segundo simulador primera parte
Segundo simulador primera parte
Segundo simulador primera parte
Segundo simulador primera parte
Segundo simulador primera parte
Segundo simulador primera parte
Segundo simulador primera parte
Segundo simulador primera parte
Segundo simulador primera parte
Segundo simulador primera parte
Segundo simulador primera parte
Segundo simulador primera parte
Segundo simulador primera parte
Segundo simulador primera parte
Segundo simulador primera parte
Segundo simulador primera parte
Segundo simulador primera parte
Segundo simulador primera parte
Segundo simulador primera parte
Segundo simulador primera parte
Segundo simulador primera parte
Segundo simulador primera parte
Segundo simulador primera parte
Segundo simulador primera parte
Segundo simulador primera parte
Segundo simulador primera parte
Segundo simulador primera parte
Segundo simulador primera parte
Segundo simulador primera parte
Segundo simulador primera parte
Segundo simulador primera parte
Segundo simulador primera parte
Segundo simulador primera parte
Segundo simulador primera parte
Segundo simulador primera parte
Segundo simulador primera parte
Segundo simulador primera parte
Segundo simulador primera parte
Segundo simulador primera parte
Segundo simulador primera parte
Segundo simulador primera parte
Segundo simulador primera parte
Segundo simulador primera parte
Segundo simulador primera parte
Segundo simulador primera parte
Segundo simulador primera parte
Segundo simulador primera parte
Segundo simulador primera parte
Segundo simulador primera parte
Segundo simulador primera parte
Segundo simulador primera parte
Segundo simulador primera parte
Segundo simulador primera parte
Segundo simulador primera parte
Segundo simulador primera parte
Segundo simulador primera parte
Segundo simulador primera parte
Segundo simulador primera parte
Segundo simulador primera parte
Segundo simulador primera parte

More Related Content

What's hot

3. Nephrotic Syndrome
3. Nephrotic Syndrome3. Nephrotic Syndrome
3. Nephrotic SyndromeWhiteraven68
 
196894071 final-case-study-pcap-docx
196894071 final-case-study-pcap-docx196894071 final-case-study-pcap-docx
196894071 final-case-study-pcap-docxhomeworkping3
 
Mumps in children 2021
Mumps in children 2021Mumps in children 2021
Mumps in children 2021Imran Iqbal
 
Case presentation on Neonatal Apnea
Case presentation on Neonatal ApneaCase presentation on Neonatal Apnea
Case presentation on Neonatal ApneaNEHA MALIK
 
Case presentation on mengoencephalitis |Inflammation of the brain
Case presentation on mengoencephalitis |Inflammation of the brain Case presentation on mengoencephalitis |Inflammation of the brain
Case presentation on mengoencephalitis |Inflammation of the brain NEHA MALIK
 
Pediatrics cases by DNB NATboard
Pediatrics  cases by DNB NATboardPediatrics  cases by DNB NATboard
Pediatrics cases by DNB NATboardRaghavendra Babu
 
Pit falls in paed practice
Pit falls in paed practicePit falls in paed practice
Pit falls in paed practiceAvinash Bhondwe
 
Paediatric nursing care plan example
Paediatric nursing care plan examplePaediatric nursing care plan example
Paediatric nursing care plan exampleExpert Writing Help
 
100 cases-in-paediatrics.....dr .Ahmed Abdallah
100 cases-in-paediatrics.....dr .Ahmed Abdallah100 cases-in-paediatrics.....dr .Ahmed Abdallah
100 cases-in-paediatrics.....dr .Ahmed Abdallahahmed Abdallah
 
8. Acute Lymphoblastic Leukemia
8. Acute Lymphoblastic Leukemia8. Acute Lymphoblastic Leukemia
8. Acute Lymphoblastic LeukemiaWhiteraven68
 
81923435 dengue-breakbone-fever-case-study
81923435 dengue-breakbone-fever-case-study81923435 dengue-breakbone-fever-case-study
81923435 dengue-breakbone-fever-case-studyhomeworkping3
 
Down syndrome case presentation pediatrics
Down syndrome case presentation pediatricsDown syndrome case presentation pediatrics
Down syndrome case presentation pediatricsShaliniShanmugam5
 
a case presentation on Acute bronchopneumonia
 a case presentation on Acute bronchopneumonia a case presentation on Acute bronchopneumonia
a case presentation on Acute bronchopneumoniaAnvy Anvia
 

What's hot (20)

Case study of neonatal jaundice
Case study of neonatal jaundiceCase study of neonatal jaundice
Case study of neonatal jaundice
 
3. Nephrotic Syndrome
3. Nephrotic Syndrome3. Nephrotic Syndrome
3. Nephrotic Syndrome
 
10. asthma
10. asthma10. asthma
10. asthma
 
196894071 final-case-study-pcap-docx
196894071 final-case-study-pcap-docx196894071 final-case-study-pcap-docx
196894071 final-case-study-pcap-docx
 
Mumps in children 2021
Mumps in children 2021Mumps in children 2021
Mumps in children 2021
 
Case presentation on Neonatal Apnea
Case presentation on Neonatal ApneaCase presentation on Neonatal Apnea
Case presentation on Neonatal Apnea
 
Case presentation on mengoencephalitis |Inflammation of the brain
Case presentation on mengoencephalitis |Inflammation of the brain Case presentation on mengoencephalitis |Inflammation of the brain
Case presentation on mengoencephalitis |Inflammation of the brain
 
Pediatrics cases by DNB NATboard
Pediatrics  cases by DNB NATboardPediatrics  cases by DNB NATboard
Pediatrics cases by DNB NATboard
 
Pit falls in paed practice
Pit falls in paed practicePit falls in paed practice
Pit falls in paed practice
 
Paediatric nursing care plan example
Paediatric nursing care plan examplePaediatric nursing care plan example
Paediatric nursing care plan example
 
Fever in children
Fever in childrenFever in children
Fever in children
 
100 cases-in-paediatrics.....dr .Ahmed Abdallah
100 cases-in-paediatrics.....dr .Ahmed Abdallah100 cases-in-paediatrics.....dr .Ahmed Abdallah
100 cases-in-paediatrics.....dr .Ahmed Abdallah
 
5. PDA
5. PDA5. PDA
5. PDA
 
8. Acute Lymphoblastic Leukemia
8. Acute Lymphoblastic Leukemia8. Acute Lymphoblastic Leukemia
8. Acute Lymphoblastic Leukemia
 
81923435 dengue-breakbone-fever-case-study
81923435 dengue-breakbone-fever-case-study81923435 dengue-breakbone-fever-case-study
81923435 dengue-breakbone-fever-case-study
 
Case Study - Cerebral Palsy
Case Study - Cerebral PalsyCase Study - Cerebral Palsy
Case Study - Cerebral Palsy
 
Down syndrome case presentation pediatrics
Down syndrome case presentation pediatricsDown syndrome case presentation pediatrics
Down syndrome case presentation pediatrics
 
PCAP-C
PCAP-CPCAP-C
PCAP-C
 
a case presentation on Acute bronchopneumonia
 a case presentation on Acute bronchopneumonia a case presentation on Acute bronchopneumonia
a case presentation on Acute bronchopneumonia
 
Kids with Bugs
Kids with BugsKids with Bugs
Kids with Bugs
 

Similar to Segundo simulador primera parte

Similar to Segundo simulador primera parte (20)

Clinical cases (2) june 9, 13
Clinical cases  (2) june 9, 13Clinical cases  (2) june 9, 13
Clinical cases (2) june 9, 13
 
Book 2009 krok 2
Book 2009 krok 2Book 2009 krok 2
Book 2009 krok 2
 
Questions 1700 - Medical MCQ without Answer
Questions 1700 - Medical MCQ without AnswerQuestions 1700 - Medical MCQ without Answer
Questions 1700 - Medical MCQ without Answer
 
Book 2006 krok-2
Book 2006 krok-2Book 2006 krok-2
Book 2006 krok-2
 
History taking a case based discussion
History taking a case based discussionHistory taking a case based discussion
History taking a case based discussion
 
Golden book for Medicine OSCE: First View
Golden book for Medicine OSCE: First ViewGolden book for Medicine OSCE: First View
Golden book for Medicine OSCE: First View
 
NIMSET 2013 Questions
NIMSET 2013 QuestionsNIMSET 2013 Questions
NIMSET 2013 Questions
 
Dermatology MCQ and AAFP.pptx
Dermatology MCQ and AAFP.pptxDermatology MCQ and AAFP.pptx
Dermatology MCQ and AAFP.pptx
 
Clinical cases (2) june 23, 13
Clinical cases  (2) june 23, 13Clinical cases  (2) june 23, 13
Clinical cases (2) june 23, 13
 
UWSA 1 2021 For USMLE Step 1 Exam
UWSA 1 2021 For USMLE Step 1 ExamUWSA 1 2021 For USMLE Step 1 Exam
UWSA 1 2021 For USMLE Step 1 Exam
 
Clinical cases (1) june 16, 13
Clinical cases  (1) june 16, 13Clinical cases  (1) june 16, 13
Clinical cases (1) june 16, 13
 
Clinical cases april 7,13
Clinical cases april 7,13Clinical cases april 7,13
Clinical cases april 7,13
 
Book 2007 krok 2
Book 2007 krok 2Book 2007 krok 2
Book 2007 krok 2
 
Clinical cases (1) june 23, 13
Clinical cases  (1) june 23, 13Clinical cases  (1) june 23, 13
Clinical cases (1) june 23, 13
 
Clinical cases (3) june 23, 13
Clinical cases  (3) june 23, 13Clinical cases  (3) june 23, 13
Clinical cases (3) june 23, 13
 
Clinical cases (3) june 23, 13
Clinical cases  (3) june 23, 13Clinical cases  (3) june 23, 13
Clinical cases (3) june 23, 13
 
Therapy 2013 krok 2
Therapy 2013 krok 2Therapy 2013 krok 2
Therapy 2013 krok 2
 
Booklet 2015 krok 2
Booklet 2015 krok 2Booklet 2015 krok 2
Booklet 2015 krok 2
 
Internal Medicine Board Review
Internal Medicine  Board ReviewInternal Medicine  Board Review
Internal Medicine Board Review
 
Gyn obs final exam, 2014
Gyn obs final exam, 2014Gyn obs final exam, 2014
Gyn obs final exam, 2014
 

More from Pharmed Solutions Institute (20)

Varicela
VaricelaVaricela
Varicela
 
Sarampion
SarampionSarampion
Sarampion
 
Rubeola
RubeolaRubeola
Rubeola
 
Quemadurasbuena
QuemadurasbuenaQuemadurasbuena
Quemadurasbuena
 
Parotidit
ParotiditParotidit
Parotidit
 
Meningitisss
MeningitisssMeningitisss
Meningitisss
 
Leucemias
LeucemiasLeucemias
Leucemias
 
Influenza
InfluenzaInfluenza
Influenza
 
Infecciones respi
Infecciones respiInfecciones respi
Infecciones respi
 
Neumonías
NeumoníasNeumonías
Neumonías
 
Hepatitis
HepatitisHepatitis
Hepatitis
 
Escarlatina
EscarlatinaEscarlatina
Escarlatina
 
Vivora
VivoraVivora
Vivora
 
Araña
ArañaAraña
Araña
 
Alacraan
AlacraanAlacraan
Alacraan
 
Parasitos
ParasitosParasitos
Parasitos
 
Diarrea
DiarreaDiarrea
Diarrea
 
Vih sida
Vih sidaVih sida
Vih sida
 
Tuberculosis
TuberculosisTuberculosis
Tuberculosis
 
Síndrome de sjogren
Síndrome de sjogrenSíndrome de sjogren
Síndrome de sjogren
 

Recently uploaded

Bhawanipatna Call Girls 📞9332606886 Call Girls in Bhawanipatna Escorts servic...
Bhawanipatna Call Girls 📞9332606886 Call Girls in Bhawanipatna Escorts servic...Bhawanipatna Call Girls 📞9332606886 Call Girls in Bhawanipatna Escorts servic...
Bhawanipatna Call Girls 📞9332606886 Call Girls in Bhawanipatna Escorts servic...Dipal Arora
 
💚Chandigarh Call Girls Service 💯Piya 📲🔝8868886958🔝Call Girls In Chandigarh No...
💚Chandigarh Call Girls Service 💯Piya 📲🔝8868886958🔝Call Girls In Chandigarh No...💚Chandigarh Call Girls Service 💯Piya 📲🔝8868886958🔝Call Girls In Chandigarh No...
💚Chandigarh Call Girls Service 💯Piya 📲🔝8868886958🔝Call Girls In Chandigarh No...Sheetaleventcompany
 
Exclusive Call Girls Bangalore {7304373326} ❤️VVIP POOJA Call Girls in Bangal...
Exclusive Call Girls Bangalore {7304373326} ❤️VVIP POOJA Call Girls in Bangal...Exclusive Call Girls Bangalore {7304373326} ❤️VVIP POOJA Call Girls in Bangal...
Exclusive Call Girls Bangalore {7304373326} ❤️VVIP POOJA Call Girls in Bangal...Sheetaleventcompany
 
Call Girl in Chennai | Whatsapp No 📞 7427069034 📞 VIP Escorts Service Availab...
Call Girl in Chennai | Whatsapp No 📞 7427069034 📞 VIP Escorts Service Availab...Call Girl in Chennai | Whatsapp No 📞 7427069034 📞 VIP Escorts Service Availab...
Call Girl in Chennai | Whatsapp No 📞 7427069034 📞 VIP Escorts Service Availab...amritaverma53
 
Call Girls in Lucknow Just Call 👉👉8630512678 Top Class Call Girl Service Avai...
Call Girls in Lucknow Just Call 👉👉8630512678 Top Class Call Girl Service Avai...Call Girls in Lucknow Just Call 👉👉8630512678 Top Class Call Girl Service Avai...
Call Girls in Lucknow Just Call 👉👉8630512678 Top Class Call Girl Service Avai...soniyagrag336
 
Chandigarh Call Girls Service ❤️🍑 9809698092 👄🫦Independent Escort Service Cha...
Chandigarh Call Girls Service ❤️🍑 9809698092 👄🫦Independent Escort Service Cha...Chandigarh Call Girls Service ❤️🍑 9809698092 👄🫦Independent Escort Service Cha...
Chandigarh Call Girls Service ❤️🍑 9809698092 👄🫦Independent Escort Service Cha...Sheetaleventcompany
 
💰Call Girl In Bangalore☎️7304373326💰 Call Girl service in Bangalore☎️Bangalor...
💰Call Girl In Bangalore☎️7304373326💰 Call Girl service in Bangalore☎️Bangalor...💰Call Girl In Bangalore☎️7304373326💰 Call Girl service in Bangalore☎️Bangalor...
💰Call Girl In Bangalore☎️7304373326💰 Call Girl service in Bangalore☎️Bangalor...Sheetaleventcompany
 
Chennai ❣️ Call Girl 6378878445 Call Girls in Chennai Escort service book now
Chennai ❣️ Call Girl 6378878445 Call Girls in Chennai Escort service book nowChennai ❣️ Call Girl 6378878445 Call Girls in Chennai Escort service book now
Chennai ❣️ Call Girl 6378878445 Call Girls in Chennai Escort service book nowtanudubay92
 
Race Course Road } Book Call Girls in Bangalore | Whatsapp No 6378878445 VIP ...
Race Course Road } Book Call Girls in Bangalore | Whatsapp No 6378878445 VIP ...Race Course Road } Book Call Girls in Bangalore | Whatsapp No 6378878445 VIP ...
Race Course Road } Book Call Girls in Bangalore | Whatsapp No 6378878445 VIP ...dishamehta3332
 
Difference Between Skeletal Smooth and Cardiac Muscles
Difference Between Skeletal Smooth and Cardiac MusclesDifference Between Skeletal Smooth and Cardiac Muscles
Difference Between Skeletal Smooth and Cardiac MusclesMedicoseAcademics
 
Dehradun Call Girls Service {8854095900} ❤️VVIP ROCKY Call Girl in Dehradun U...
Dehradun Call Girls Service {8854095900} ❤️VVIP ROCKY Call Girl in Dehradun U...Dehradun Call Girls Service {8854095900} ❤️VVIP ROCKY Call Girl in Dehradun U...
Dehradun Call Girls Service {8854095900} ❤️VVIP ROCKY Call Girl in Dehradun U...Sheetaleventcompany
 
tongue disease lecture Dr Assadawy legacy
tongue disease lecture Dr Assadawy legacytongue disease lecture Dr Assadawy legacy
tongue disease lecture Dr Assadawy legacyDrMohamed Assadawy
 
(RIYA)🎄Airhostess Call Girl Jaipur Call Now 8445551418 Premium Collection Of ...
(RIYA)🎄Airhostess Call Girl Jaipur Call Now 8445551418 Premium Collection Of ...(RIYA)🎄Airhostess Call Girl Jaipur Call Now 8445551418 Premium Collection Of ...
(RIYA)🎄Airhostess Call Girl Jaipur Call Now 8445551418 Premium Collection Of ...TanyaAhuja34
 
Chandigarh Call Girls Service ❤️🍑 9809698092 👄🫦Independent Escort Service Cha...
Chandigarh Call Girls Service ❤️🍑 9809698092 👄🫦Independent Escort Service Cha...Chandigarh Call Girls Service ❤️🍑 9809698092 👄🫦Independent Escort Service Cha...
Chandigarh Call Girls Service ❤️🍑 9809698092 👄🫦Independent Escort Service Cha...Sheetaleventcompany
 
Kolkata Call Girls Shobhabazar 💯Call Us 🔝 8005736733 🔝 💃 Top Class Call Gir...
Kolkata Call Girls Shobhabazar  💯Call Us 🔝 8005736733 🔝 💃  Top Class Call Gir...Kolkata Call Girls Shobhabazar  💯Call Us 🔝 8005736733 🔝 💃  Top Class Call Gir...
Kolkata Call Girls Shobhabazar 💯Call Us 🔝 8005736733 🔝 💃 Top Class Call Gir...Namrata Singh
 
Call Girls Kathua Just Call 8250077686 Top Class Call Girl Service Available
Call Girls Kathua Just Call 8250077686 Top Class Call Girl Service AvailableCall Girls Kathua Just Call 8250077686 Top Class Call Girl Service Available
Call Girls Kathua Just Call 8250077686 Top Class Call Girl Service AvailableDipal Arora
 
❤️Call Girl Service In Chandigarh☎️9814379184☎️ Call Girl in Chandigarh☎️ Cha...
❤️Call Girl Service In Chandigarh☎️9814379184☎️ Call Girl in Chandigarh☎️ Cha...❤️Call Girl Service In Chandigarh☎️9814379184☎️ Call Girl in Chandigarh☎️ Cha...
❤️Call Girl Service In Chandigarh☎️9814379184☎️ Call Girl in Chandigarh☎️ Cha...Sheetaleventcompany
 
Pune Call Girl Service 📞9xx000xx09📞Just Call Divya📲 Call Girl In Pune No💰Adva...
Pune Call Girl Service 📞9xx000xx09📞Just Call Divya📲 Call Girl In Pune No💰Adva...Pune Call Girl Service 📞9xx000xx09📞Just Call Divya📲 Call Girl In Pune No💰Adva...
Pune Call Girl Service 📞9xx000xx09📞Just Call Divya📲 Call Girl In Pune No💰Adva...Sheetaleventcompany
 
7 steps How to prevent Thalassemia : Dr Sharda Jain & Vandana Gupta
7 steps How to prevent Thalassemia : Dr Sharda Jain & Vandana Gupta7 steps How to prevent Thalassemia : Dr Sharda Jain & Vandana Gupta
7 steps How to prevent Thalassemia : Dr Sharda Jain & Vandana GuptaLifecare Centre
 
Call Girl In Indore 📞9235973566📞 Just📲 Call Inaaya Indore Call Girls Service ...
Call Girl In Indore 📞9235973566📞 Just📲 Call Inaaya Indore Call Girls Service ...Call Girl In Indore 📞9235973566📞 Just📲 Call Inaaya Indore Call Girls Service ...
Call Girl In Indore 📞9235973566📞 Just📲 Call Inaaya Indore Call Girls Service ...Sheetaleventcompany
 

Recently uploaded (20)

Bhawanipatna Call Girls 📞9332606886 Call Girls in Bhawanipatna Escorts servic...
Bhawanipatna Call Girls 📞9332606886 Call Girls in Bhawanipatna Escorts servic...Bhawanipatna Call Girls 📞9332606886 Call Girls in Bhawanipatna Escorts servic...
Bhawanipatna Call Girls 📞9332606886 Call Girls in Bhawanipatna Escorts servic...
 
💚Chandigarh Call Girls Service 💯Piya 📲🔝8868886958🔝Call Girls In Chandigarh No...
💚Chandigarh Call Girls Service 💯Piya 📲🔝8868886958🔝Call Girls In Chandigarh No...💚Chandigarh Call Girls Service 💯Piya 📲🔝8868886958🔝Call Girls In Chandigarh No...
💚Chandigarh Call Girls Service 💯Piya 📲🔝8868886958🔝Call Girls In Chandigarh No...
 
Exclusive Call Girls Bangalore {7304373326} ❤️VVIP POOJA Call Girls in Bangal...
Exclusive Call Girls Bangalore {7304373326} ❤️VVIP POOJA Call Girls in Bangal...Exclusive Call Girls Bangalore {7304373326} ❤️VVIP POOJA Call Girls in Bangal...
Exclusive Call Girls Bangalore {7304373326} ❤️VVIP POOJA Call Girls in Bangal...
 
Call Girl in Chennai | Whatsapp No 📞 7427069034 📞 VIP Escorts Service Availab...
Call Girl in Chennai | Whatsapp No 📞 7427069034 📞 VIP Escorts Service Availab...Call Girl in Chennai | Whatsapp No 📞 7427069034 📞 VIP Escorts Service Availab...
Call Girl in Chennai | Whatsapp No 📞 7427069034 📞 VIP Escorts Service Availab...
 
Call Girls in Lucknow Just Call 👉👉8630512678 Top Class Call Girl Service Avai...
Call Girls in Lucknow Just Call 👉👉8630512678 Top Class Call Girl Service Avai...Call Girls in Lucknow Just Call 👉👉8630512678 Top Class Call Girl Service Avai...
Call Girls in Lucknow Just Call 👉👉8630512678 Top Class Call Girl Service Avai...
 
Chandigarh Call Girls Service ❤️🍑 9809698092 👄🫦Independent Escort Service Cha...
Chandigarh Call Girls Service ❤️🍑 9809698092 👄🫦Independent Escort Service Cha...Chandigarh Call Girls Service ❤️🍑 9809698092 👄🫦Independent Escort Service Cha...
Chandigarh Call Girls Service ❤️🍑 9809698092 👄🫦Independent Escort Service Cha...
 
💰Call Girl In Bangalore☎️7304373326💰 Call Girl service in Bangalore☎️Bangalor...
💰Call Girl In Bangalore☎️7304373326💰 Call Girl service in Bangalore☎️Bangalor...💰Call Girl In Bangalore☎️7304373326💰 Call Girl service in Bangalore☎️Bangalor...
💰Call Girl In Bangalore☎️7304373326💰 Call Girl service in Bangalore☎️Bangalor...
 
Chennai ❣️ Call Girl 6378878445 Call Girls in Chennai Escort service book now
Chennai ❣️ Call Girl 6378878445 Call Girls in Chennai Escort service book nowChennai ❣️ Call Girl 6378878445 Call Girls in Chennai Escort service book now
Chennai ❣️ Call Girl 6378878445 Call Girls in Chennai Escort service book now
 
Race Course Road } Book Call Girls in Bangalore | Whatsapp No 6378878445 VIP ...
Race Course Road } Book Call Girls in Bangalore | Whatsapp No 6378878445 VIP ...Race Course Road } Book Call Girls in Bangalore | Whatsapp No 6378878445 VIP ...
Race Course Road } Book Call Girls in Bangalore | Whatsapp No 6378878445 VIP ...
 
Difference Between Skeletal Smooth and Cardiac Muscles
Difference Between Skeletal Smooth and Cardiac MusclesDifference Between Skeletal Smooth and Cardiac Muscles
Difference Between Skeletal Smooth and Cardiac Muscles
 
Dehradun Call Girls Service {8854095900} ❤️VVIP ROCKY Call Girl in Dehradun U...
Dehradun Call Girls Service {8854095900} ❤️VVIP ROCKY Call Girl in Dehradun U...Dehradun Call Girls Service {8854095900} ❤️VVIP ROCKY Call Girl in Dehradun U...
Dehradun Call Girls Service {8854095900} ❤️VVIP ROCKY Call Girl in Dehradun U...
 
tongue disease lecture Dr Assadawy legacy
tongue disease lecture Dr Assadawy legacytongue disease lecture Dr Assadawy legacy
tongue disease lecture Dr Assadawy legacy
 
(RIYA)🎄Airhostess Call Girl Jaipur Call Now 8445551418 Premium Collection Of ...
(RIYA)🎄Airhostess Call Girl Jaipur Call Now 8445551418 Premium Collection Of ...(RIYA)🎄Airhostess Call Girl Jaipur Call Now 8445551418 Premium Collection Of ...
(RIYA)🎄Airhostess Call Girl Jaipur Call Now 8445551418 Premium Collection Of ...
 
Chandigarh Call Girls Service ❤️🍑 9809698092 👄🫦Independent Escort Service Cha...
Chandigarh Call Girls Service ❤️🍑 9809698092 👄🫦Independent Escort Service Cha...Chandigarh Call Girls Service ❤️🍑 9809698092 👄🫦Independent Escort Service Cha...
Chandigarh Call Girls Service ❤️🍑 9809698092 👄🫦Independent Escort Service Cha...
 
Kolkata Call Girls Shobhabazar 💯Call Us 🔝 8005736733 🔝 💃 Top Class Call Gir...
Kolkata Call Girls Shobhabazar  💯Call Us 🔝 8005736733 🔝 💃  Top Class Call Gir...Kolkata Call Girls Shobhabazar  💯Call Us 🔝 8005736733 🔝 💃  Top Class Call Gir...
Kolkata Call Girls Shobhabazar 💯Call Us 🔝 8005736733 🔝 💃 Top Class Call Gir...
 
Call Girls Kathua Just Call 8250077686 Top Class Call Girl Service Available
Call Girls Kathua Just Call 8250077686 Top Class Call Girl Service AvailableCall Girls Kathua Just Call 8250077686 Top Class Call Girl Service Available
Call Girls Kathua Just Call 8250077686 Top Class Call Girl Service Available
 
❤️Call Girl Service In Chandigarh☎️9814379184☎️ Call Girl in Chandigarh☎️ Cha...
❤️Call Girl Service In Chandigarh☎️9814379184☎️ Call Girl in Chandigarh☎️ Cha...❤️Call Girl Service In Chandigarh☎️9814379184☎️ Call Girl in Chandigarh☎️ Cha...
❤️Call Girl Service In Chandigarh☎️9814379184☎️ Call Girl in Chandigarh☎️ Cha...
 
Pune Call Girl Service 📞9xx000xx09📞Just Call Divya📲 Call Girl In Pune No💰Adva...
Pune Call Girl Service 📞9xx000xx09📞Just Call Divya📲 Call Girl In Pune No💰Adva...Pune Call Girl Service 📞9xx000xx09📞Just Call Divya📲 Call Girl In Pune No💰Adva...
Pune Call Girl Service 📞9xx000xx09📞Just Call Divya📲 Call Girl In Pune No💰Adva...
 
7 steps How to prevent Thalassemia : Dr Sharda Jain & Vandana Gupta
7 steps How to prevent Thalassemia : Dr Sharda Jain & Vandana Gupta7 steps How to prevent Thalassemia : Dr Sharda Jain & Vandana Gupta
7 steps How to prevent Thalassemia : Dr Sharda Jain & Vandana Gupta
 
Call Girl In Indore 📞9235973566📞 Just📲 Call Inaaya Indore Call Girls Service ...
Call Girl In Indore 📞9235973566📞 Just📲 Call Inaaya Indore Call Girls Service ...Call Girl In Indore 📞9235973566📞 Just📲 Call Inaaya Indore Call Girls Service ...
Call Girl In Indore 📞9235973566📞 Just📲 Call Inaaya Indore Call Girls Service ...
 

Segundo simulador primera parte

  • 1. SEGUNDO SIMULADOR DEL CURSO ENARM CMN SIGLO XXI PRIMERA PARTE
  • 2. Instrucciones 1. Preparar hoja de respuesta impresa. 2. Activar el simulador aplicando play. 3. Escribir nombre y hora de inicio en la hoja de respuesta. 4. Circular el reactivo de cada respuesta en hoja de respuesta. 5. El simulador se activará a las 8:00 hrs y se desactivará a las 14:00 hrs, la primera parte. 6. La segunda parte se activara a las 16:00 hrs y de desactivará a las 20:00 hrs.
  • 3. Caso clínico No. 58 (ejemplo) Femenino de 51 años, originaria de Campeche, acude control mensual por HAS y DM 10 y 5 años respectivamente, sus SV fueron TA 155/95 mmHg, FC 51 lpm, FR 21 rpm. Glucosa 210 mg/dl, hemoglobina glucosilada 6, urea 17, creatinina 1,1 mg/dl, refiere mareo ocasional, cefalea global acompañada de sueño y fatiga, frio y mareo ocasional, disnea de leves a moderados esfuerzos, se observa edema de miembros inferiores. Su tratamiento es captopril, metoprolol y glibenclamida, se realiza ECG (ver imagen).
  • 4. Caso clínico No. 58 (ejemplo)
  • 5. Caso clínico No. 58 (ejemplo) PRENGUNTA Se realiza ECG, considerando las manifestaciones que se han presentado, que conducta terapéutica considera más apropiada realizar de forma mediata? RESPUESTA a.- Incrementar los fármacos hasta meta de <120/80 mmHg. b.- Retirar metoprolol continuar con captopril y glibenclamida. c.- Mantener glibenclamida, retirar metoprolol e iniciar losartan. d.- Agregar hidroclorotiazida al manejo actual
  • 6. Hoja de respuestas d 51 a b c d 101 d 52 a b c d 102 d 53 a b c d 103 d 54 a b c d 104 d 55 a b c d 105 d 56 a b c d 106 d 57 a b c d 107 d 58 a b c d 108 d 59 a b c d 109 d 60 a b c d 110
  • 8. CASE REPORT A recently married 29-year-old nulliparous African- American female presents with uncomplicated cystitis. She is otherwise healthy. She reports that she is currently using barrier birth control methods but plans to discontinue this soon, as she would like to become pregnant.
  • 9. QUESTION No. 1 Which one of the following supplemental vitamins or minerals would you advise this patient to begin taking? a) Thiamine b) Iodine c) Folate d) Calcium
  • 10. QUESTION No. 2 Which one of the following is true regarding testing the rotator cuff for injury? ANSWER a) The subscapularis is tested with lateral rotation against resistance b) The teres minor is tested with medial rotation against resistance c) The infraspinatus is tested with medial rotation against resistance d) The supraspinatus is tested with abduction against resistance
  • 11. CASE REPORT You have recently begun caring for a 25-year-old white female who has multiple complaints. You have seen her 3 times for walk-in office visits over the past month. She has shown appreciation for your work during the encounters, but has been critical of your care when talking to the office staff. At times she has been kind and charming, and at other times she has been rude and verbally abusive to your staff. She has a string of multiple relationships in the past, none of which has lasted very long. During times of intense stress, she has sometimes engaged in self-mutilation. She frequently changes jobs and living arrangements.
  • 12. QUESTION No. 3 Which one of the following strategies would be most appropriate in the care of this patient ANSWER a) Strive to develop a close relationship with the patient. b) Ignore verbal attacks on staff members. c) Provide detailed, technical explanations for. any therapies provided. d) Schedule frequent office visits for follow-up.
  • 13. In an 11-year-old male with dark brown urine and hand and foot edema. QUESTION No. 4 Which one of the following would be most suggestive of glomerulonephritis? ANSWER a) RBC casts in the urine b) Elevated C3 and C4 complement levels c) Eosinophils in the urine d) Positive serum antinuclear antibody levels
  • 14. A 32-year-old white female at 16 weeks gestation presents to your office with right lower quadrant pain. QUESTION No. 5 Which one of the following imaging studies would be most appropriate for initial evaluation of this patient? ANSWER a) MRI of the Abdomen b) Intravenous pyelography c) CT of the abdomen d) Ultrasonography of the abdomen
  • 15. QUESTION No. 6 Which one of the following is the leading cause of blindness in individuals over age 65? a) Narrow angle glaucoma b) Diabetic retinopathy c) Macular degeneration d) Ophthalmic artery occlusion
  • 16. CASE REPORT A 48-year-old white female complains of anxiety and difficulty concentrating at home and at work. She reports that the symptoms have increased over the last 2 months because of her daughter’s marital difficulties. She has had similar symptoms along with intermittent depression since she was a teenager. She admits to a loss of pleasure in work and recreational activities.
  • 17. QUESTION No. 7 Which one of the following is LEAST likely to help her coexistent depressive symptoms? ANSWER a) Nortriptyline. b) Buspirone. c) Escitalopram d) Venlafaxine
  • 18. CASE REPORT An 82-year-old white female visits your office. She reports a 2- week history of shoulder stiffness that is severe in the morning, and a 3-day history of pain on the left side of the face while chewing food. She denies dental pain or sensitivity. Her family history is negative for rheumatologic disorders. Your evaluation reveals a 5-lb weight loss and evidence of mild depression. Her temperature is 38.0° C (100.4° F), she has no dental decay or carotid bruit, and her left temporal scalp is tender. Her lungs are clear to auscultation, there are no abnormal heart sounds, and her abdomen is nontender. She has no joint swelling or warmth, but experiences discomfort with shoulder and hip range of motion. Laboratory analysis reveals a hemoglobin level of 11.0 g/dL (N 11.7–16.1), an erythrocyte sedimentation rate of 80 mm/hr (N 0–30), a serum glucose level of 120 mg/dL, and a BUN level of 24 mg/dL (N 8–23).
  • 19. QUESTION No. 8 Which one of the following is the greatest immediate risk for this patient? ANSWER a) Monocular vision loss b) Hemiparesis c) Sudden death d) Adhesive capsulitis of the shoulder
  • 20. CASE REPORT A 73-year-old male with COPD presents to the emergency department with increasing dyspnea. Examination reveals no sign of jugular venous distention. A chest examination reveals decreased breath sounds and scattered rhonchi, and the heart sounds are very distant but no gallop or murmur is noted. There is +1 edema of the lower extremities. Chest radiographs reveal cardiomegaly but no pleural effusion. The patient’s B-type natriuretic peptide level is 850 pg/mL (N <100) and his serum creatinine level is 0.8 mg/dL (N 0.6–1.5).
  • 21. QUESTION No. 9 Which one of the following would be the most appropriate initial management? ANSWER a) Prednisone, 20 mg twice daily for 1 week b) Furosemide, 40 mg intravenously c) Intravenous heparin d) Tiotropium
  • 22. CASE REPORT A 28-year-old primigravida is at 20 weeks gestation by dates but her fundal height is consistent with a 26- week gestation. She has had episodes of vomiting during the pregnancy that were more severe than the physiologic vomiting typically seen in pregnancy. A sonogram performed at about 5 weeks gestation for vaginal bleeding was normal and showed a single fetus.
  • 23. QUESTION No. 10 Which one of the following would be most appropriate at this point? ANSWER a) Expectant management b) A repeat sonogram c) MRI of the pelvis d) A serum hCG leve
  • 24. CASE REPORT A 24-year-old male, new to your practice, presents for a mental health evaluation. The patient has a past history of schizophrenia, diagnosed several years ago. QUESTION No. 11 Which one of the following, if present, would lead to a reconsideration of this diagnosis? ANSWER a) Loose associations b) Elated mood c) Social dysfunction d) Auditory hallucinations
  • 25. CASE REPORT A previously healthy 22-year-old female presents for her regular prenatal checkup at 38 weeks gestation. She has a blood pressure of 145/95 mm Hg today and this is unchanged 1 hour later. Her blood pressure was normal before pregnancy. She is otherwise feeling well. She has moderate edema at the ankles and 3+ reflexes at the knees and ankles. A urinalysis for protein is normal.
  • 26. QUESTION No. 12 Given this presentation, which one of the following is the most likely diagnosis? ANSWER a) Hemolysis, elevated liver enzymes, low platelets syndrome b) Preeclampsia c) Essential hypertension d) Gestational hypertension
  • 27. CASE REPORT A 72-year-old female with longstanding diabetes mellitus presents to your office. During the review of systems, she complains of difficulty voiding and frequent “dribbling.” A urinalysis is negative for infection and her post-void residual volume is 250 mL.
  • 28. QUESTION No. 13 Which one of the following is the most likely cause of this patient’s urinary incontinence? ANSWER a) A grade II cystocele b) Atrophic vaginitis c) Asymptomatic bacteriuria d) Autonomic neuropathy
  • 29. CASE REPORT A 3-year-old male is brought to your office the day after he was stung by a honeybee. He has developed a significant local reaction, with redness and swelling around the site of the sting on his forearm, and also has had some swelling of his lips for “a couple of hours.” His mother removed the stinger and gave him some oral diphenhydramine The local reaction has now almost resolved and he has not had any hives or respiratory distress.
  • 30. QUESTION No. 14 Which one of the following is true concerning this situation? ANSWER a) An antibiotic that covers Streptococcus pyogenes and Streptococcus aureus should be administered b) If venom immunotherapy is begun, it should be stopped after 2 years of treatment c) The child is at high risk for a systemic reaction if he is stung again in the future d) Skin tests are not helpful in confirming the presence of insect sting allergy
  • 31. CASE REPORT A 65-year-old white female develops a burning pain in the left lateral thorax, followed 2 days later by an erythematous vesicular rash. QUESTION No. 15 Of the following, the best treatment is? ANSWER a) topical capsaicin b) oral corticosteroids c) topical corticosteroids d) topical acyclovir
  • 32. QUESTION No. 16 Which one of the following would be most appropriate to treat a dental infection requiring antibiotic therapy? ANSWER a) Penicillin b) Dicloxacillin c) Erythromycin d) Tetracycline
  • 33. CASE REPORT You are consulted for medical management of a 45- year-old male, previously unknown to you, who is hospitalized in the psychiatric unit with paranoid schizophrenia. His fasting blood glucose level is 180 mg/dL.
  • 34. QUESTION 17 Which one of the following medications is the most likely cause of the hyperglycemia? ANSWER a) Alprazolam b) Chlorpromazine c) Olanzapine d) Haloperidol
  • 35. CASE REPORT A 22-year-old male presents to the emergency department several hours after a rugby match in which he was struck in the face. His nose is bleeding. On examination the patient has a depression in his nose on the side of impact and an outward displacement on the opposite side. Internal inspection reveals a purple area of swelling and fluctuance on the left side of the nasal septum. The remainder of the examination demonstrates no findings to suggest other facial or periorbital trauma.
  • 36. QUESTION 18 Which one of the following would be most appropriate at this time? ANSWER a) Immediate CT imaging to assess for facial and mandibular fracture b) Incision and drainage of the swollen, fluctuant area c) Immediate plain radiographs of the nose and orbits d) Immediate closed reduction of the nasal fracture
  • 37. CASE REPORT A 30-year-old gravida 3 para 2 at 28 weeks gestation is a restrained passenger in a high-speed motor vehicle accident. After initial stabilization in the field with supplemental oxygen and intravenous fluids, she is brought into the emergency department on a backboard and wearing a cervical collar.
  • 38. CASE REPORT Your hospital administrator asks you to develop a community screening program for melanoma. QUESTION 19 Which one of the following is true concerning screening for this disease? ANSWER a) Screening for melanoma is not indicated since screening takes too much time b) No definite clinical evidence has shown that screening for melanoma reduces mortality c) Screening for melanoma is not indicated since the disease is rare d) Because of sunbathing, female patients are the most important population to screen
  • 39. CASE REPORT A 19-year-old white female presents for an initial family planning evaluation. Specifically, she is interested in oral contraception. She is not presently sexually active, but has a steady boyfriend. She has no contraindications to oral contraceptive use. She has mild acne vulgaris. You discuss possible side effects and benefits of combined oral contraceptives, including improvement of her acne.
  • 40. QUESTION 20 Which one of the following is also associated with oral contraceptive use? ANSWER a) Increased incidence of dysmenorrhea b) Increased risk of ovarian cancer c) Increased risk for ectopic pregnancy d) Decreased risk of ovarian cysts
  • 41. CASE REPORT A 78-year-old white female notices a scant milky secretion from her breast. QUESTION 21 Which one of the following medications can cause this symptom? ANSWER a) Diazepam b) Risperidone c) Famotidine d) Zaleplon
  • 42. CASE REPORT You have just diagnosed and treated gonorrheal cervicitis in a 24-year-old female who is in her second trimester of pregnancy. The patient has a friend who had a stillborn infant and she is concerned that the gonorrhea may predispose her to stillbirth.
  • 43. QUESTION 22 Which one of the following would be most appropriate in this situation? ANSWER a) Perform monthly vaginal cultures for gonorrhea, starting at 24 weeks gestation b) Perform fetal monitoring and serial vaginal cultures, starting at 32 weeks gestation c) Inform the patient that stillbirth related to gonorrhea is very rare, and that special fetal and maternal monitoring is not needed d) Administer prophylactic ciprofloxacin (Cipro) weekly until delivery
  • 44. CASE REPORT A 6-year-old male is brought in for evaluation by his mother, who is concerned that he may have asthma. She reports that he coughs about 3 days out of the week and has a nighttime cough approximately 1 night per week. There is a family history of eczema and allergic rhinitis.
  • 45. QUESTION 23 Which one of the following would be the preferred initial treatment for this patient? ANSWER a) A mast-cell stabilizer such as cromolyn sodium b) A leukotriene receptor antagonist such as montelukast c) A long-acting beta-agonist such as salmeterol d) A low-dose inhaled corticosteroid such as budesonide
  • 46. QUESTION 24 Which one of the following is the most correct recommendation regarding seat belt use for a woman at 38 weeks gestation? ANSWER a) The seat belt should be positioned over the dome of the uterus and the shoulder harness should be positioned between the breasts b) The seat belt should be positioned under the abdomen over both the anterior superioriliac spines and the pubic symphysis; the belt should be applied with some slack c) The seat belt should be positioned under the abdomen over both the anterior superior iliac spines and the pubic symphysis; the shoulder harness should be positioned between the breasts; the belt should be applied as snugly as comfort will allow d) Seat belts should not be used in the later stages of pregnancy
  • 47. CASE REPORT A 34-year-old female presents to the emergency department with a severe migraine headache unresponsive to tramadol and sumatriptan at home. She takes fluoxetine for depression. Soon after being given an injection of meperidine, she develops agitation, diaphoresis, tremor, diarrhea, fever, and incoordination.
  • 48. QUESTION 25 The most likely cause of this patient’s symptoms is? ANSWER a) serotonin syndrome b) panic attack c) thyrotoxic storm d) viral encephalitis
  • 49. QUESTION 26 The development of azotemia suggests the presence of underlying renal artery stenosis in a patient taking which one of the following? ANSWER a) Amlodipine b) Lisinopril c) Hydrochlorothiazide d) Metoprolol
  • 50. QUESTION 27 Which one of the following is a risk factor for endometrial cancer? ANSWER a) Multiparity b) Use of oral contraceptives c) Late menarche d) Polycystic ovary syndrome
  • 51. QUESTION 28 Promoting good sleep hygiene is basic in the treatment of insomnia. Which one of the following measures will aid in promoting healthy sleep habits? ANSWER a) Taking an enjoyable book or magazine to bed to read b) Eating the heaviest meal of the day close to bedtime c) Drinking a glass of wine as a sedative before retiring d) Maintaining a regular sleep/wake schedule
  • 52. QUESTION 29 Until you are able to rule out a spinal injury, in what position should the patient be kept? ANSWER a) Supine, with the uterus manually deflected laterally b) Prone c) Trendelenburg’s pattern d) Left lateral decubitus
  • 53. QUESTION A 47-year-old male presents with a history of fatigue, arthralgias, nonspecific abdominal pain, and erectile dysfunction. The initial laboratory workup reveals a normal CBC and basic metabolic profile, but slightly elevated transaminases.
  • 54. QUESTION 30 Which one of the following is the most appropriate initial test to evaluate for hereditary hemochromatosis? ANSWER a) Serum transferrin saturation b) Serum ceruloplasmin testing c) Serum alpha-fetoprotein testing d) Serum ferritin testing
  • 55. CASE REPORT A 56-year-old female presents for a health maintenance examination. She has a history of a total hysterectomy for benign disease 4 years ago. You are able to document that the hysterectomy pathology was benign and that she has had normal Papanicolaou (Pap) tests for 10 years. The patient asks about regular Pap smears.
  • 56. QUESTION 31 Which one of the following would be the most appropriate recommendation? ANSWER a) Routine Pap smears should be continued until age 70 b) A Pap smear should be done every 3 years c) A Pap smear should be done yearly for 3 years and only if indicated thereafter d) A Pap smear is not indicated
  • 57. QUESTION A 31-year-old African-American female presents with the chief complaint of bilateral galactorrhea of 3 months’ duration. She also has not menstruated for 1 year despite changing birth control pills several times. A review of systems is otherwise noncontributory. Except for a milky discharge with stimulation of the breasts, her examination is within normal limits. Serum prolactin on two occasions is >200 :g/L (N 0–20).
  • 58. QUESTION 32 Which one of the following would be most appropriate at this point? ANSWER a) Stop her oral contraceptive and repeat the serum prolactin level in 1 month b) Start the patient on risperidone c) Order bilateral mammography d) Order a brain MRI with enhancement and emphasis on the pituitary fossa
  • 59. CASE REPORT An 84-year-old African-American female is brought to your office by her daughter, who is concerned that the mother has memory problems and is neglecting to pay her monthly bills. The mother also is forgetting appointments and asks the same questions repeatedly. This problem has been steadily worsening over the last 1–2 years. The patient has very little insight into her problems, scores 24 out of a possible 30 points on the Mini-Mental State Examination, and has difficulty with short-term recall and visuospatial tasks. Her physical examination and a thorough laboratory workup are normal. A CT scan of the brain mreveals diffuse atrophy.
  • 60. QUESTION 33 Which one of the following is the most likely etiology for this patient’s memory problem? ANSWER a) Alzheimer’s disease b) Dementia resulting from depression c) Normal aging d) Lewy body dementia
  • 61. CASE REPORT A 23-year-old female is at 8 weeks gestation with her first pregnancy. She is planning to travel to Third World countries soon for job-related reasons. QUESTION 34 Which one of the following is contraindicated in this patient? ANSWER a) Hepatitis B vaccine b) Rabies vaccine c) Varicella vaccine d) Meningococcal vaccine
  • 62. QUESTION 35 In the evaluation of foot ulcerations, a neuropathic etiology is suggested by which one of the following? ANSWER a) Abnormal monofilament testing b) Absence of toe hair c) Erectile dysfunction d) Distal foot pain when supine
  • 63. QUESTION 36 A 72-year-old white male has new-onset hypertension with a current blood pressure of 190/110mm Hg. Which one of the following agents can be used as part of a test for diagnosing renovascular hypertension, but would also increase the risk for azotemia if used for treatment? ANSWER a) Furosemide b) Metoprolol c) Captopril d) Amlodipine
  • 64. QUESTION 37 Which one of the following treatments for childhood nocturnal enuresis has both the highest cure rates and the lowest relapse rates? ANSWER a) Positive reinforcement b) Imipramine c) Responsibility training d) Bed-wetting alarms
  • 65. CASE REPORT You see a 30-year-old male who has just fallen on an outstretched hand. He complains of wrist pain and edema. Examination reveals tenderness over the anatomic snuffbox and over the scaphoid tubercle at the proximal wrist crease with the hand in extension. Radiographs of the wrist are negative.
  • 66. QUESTION 38 Which one of the following would be the most appropriate action at this point? ANSWER a) Order high-spatial-resolution ultrasonography of the wrist b) Order a bone scan for the next day c) Immobilize in a thumb spica splint for 1–2 weeks and then order repeat radiographs d) Immobilize in a cast for 6–8 weeks
  • 67. CASE REPORT A 15-month-old male is brought to the emergency department following a generalized tonic-clonic seizure at home. The parents report that the seizure lasted 5 minutes, with confusion for the next 15 minutes. This is the child’s first seizure. There is no family history of seizures. His medical history is normal except for a 1-day history of a URI. While initially lethargic in the emergency department, the child is now awake and playful, with a temperature of 39.5° C (103.2° F) and a normal examination. Appropriate diagnostic tests are performed, including a blood glucose level, which is 96 mg/dL.
  • 68. QUESTION 39 Which one of the following would be most appropriate to administer at this point? ANSWER a) Ceftriaxone intravenously b) Phenobarbital orally c) Carbamazepine orally d) Acetaminophen orally
  • 69. CASE REPORT A 77-year-old white male complains of urinary incontinence of more than one year’s duration. The incontinence occurs with sudden urgency. No association with coughing or positional change has been noted. There is no history of fever or dysuria. One year ago he underwent transurethral resection of the prostate (TURP) for benign prostatic hypertrophy and says his urinary stream has improved. Rectal examination reveals a smoothly enlarged prostate without nodularity, and normal sphincter tone. No residual urine is found with post- void catheterization.
  • 70. QUESTION 40 Which one of the following is the most likely cause of this patient’s incontinence? ANSWER a) Overflow b) Urinary tract infection c) Fecal impaction d) Detrusor instability
  • 71. QUESTION 41 A health-care worker has a negative tuberculin skin test (Mantoux method). A second test 10 days later is positive. This result indicates ANSWER a) a false-positive skin test b) long-standing, latent infection c) probable immunodeficiency d) previous vaccination with BCG
  • 72. QUESTION 42 The most common manifestation of uterine rupture during labor is ANSWER a) vaginal hemorrhage b) regression of the fetus c) sudden, tearing uterine pain d) fetal distress
  • 73. QUESTION 43 Which one of the following tests is most useful for the initial workup of suspected Alzheimer’s disease? ANSWER a) Carotid Doppler ultrasonography b) An erythrocyte sedimentation rate c) A cardiac stress test d) A TSH level
  • 74. CASE REPORT A 78-year-old white female presents with a 3-day history of lower thoracic back pain. She denies any antecedent fall or trauma, and first noted the pain upon arising. Her description of the pain indicates that it is severe, bilateral, and without radiation to the arms or legs. Her past medical history is positive for hypertension and controlled diabetes mellitus. Her medications include hydrochlorothiazide, enalapril, metformin, and a general multivitamin. She is a previous smoker but does not drink alcohol. She underwent menopause at age 50 and took estrogen for “a few months” for hot flashes. Physical examination reveals her to be in moderate pain with a somewhat stooped posture and mild tenderness over T12– L1. She has negative straight-leg raising and normal lower extremity sensation, strength, and reflexes.
  • 75. QUESTION 44 Which one of the following is true regarding this patient’s likely condition? ANSWER a) Subcutaneous or intranasal calcitonin may be very helpful for pain relief b) Investigation for an underlying malignancy is indicated c) Prolonged (approximately 2 weeks) bed rest will increase the chance of complete recovery d) An MRI or nuclear medicine bone scan should be performed
  • 76. QUESTION 45 Which one of the following agents used for tocolysis has the unique adverse effect of respiratory depression? ANSWER a) Ritodrine b) Magnesium sulfate c) Terbutaline d) Indomethacin
  • 77. QUESTION 46 A 5-year-old male is scheduled for elective hernia repair at 11:00 a.m. Which one of the following would be the most appropriate recommendation? ANSWER a) No solid food for 8 hours prior to surgery and clear liquids until 2 hours prior to surgery b) No solid food after midnight and nothing by mouth 8 hours prior to surgery c) Nothing by mouth 8 hours prior to surgery d) Nothing by mouth 2 hours prior to surgery
  • 78. QUESTION 47 The only nonsexual behavior that is consistently and strongly correlated with cervical dysplasia and cervical cancer is ANSWER a) caffeine consumption b) cigarette smoking c) cocaine use d) a high-fat diet
  • 79. QUESTION 48 Which one of the following is the first-line antibiotic treatment for uncomplicated acute otitis media? ANSWER a) Ceftriaxone b) Trimethoprim/sulfamethoxazole c) Amoxicillin d) Cefuroxime
  • 80. CASE REPORT A 4-year-old male has a fever of 1 week’s duration. It has been at or slightly above 38° C (101°F) and has responded poorly to antipyretics. The patient complains of photophobia, burning in his eyes, and a sore throat. His mother also notes that his eyes look red, his lips are red and cracked, and he has a “strawberry tongue.” The child’s palms and soles are erythematous and the periungual regions show desquamation of the skin. He has minimally painful nodes located in the anterior cervical region, about 2×2 cm in size. A Streptococcus screen is negative.
  • 81. QUESTION 49 The most appropriate management at this time would be? ANSWER a) intravenous nafcillin b) intramuscular benzathine penicillin G 600,000 U c) intravenous immune globulin and aspirin d) prednisone, 2–3 mg/kg daily
  • 82. QUESTION 50 Patients with symptomatic congestive heart failure associated with a reduced systolic ejection fraction or left ventricular remodeling should be initially treated with which one of the following agents? ANSWER a) Verapamil b) An ACE inhibitor c) Hydralazine d) Warfarin
  • 83. CASE REPORT A 6-year-old white male visits your office with chief complaints of a recent onset of fever, bilateral knee and ankle pain, colicky abdominal pain, and rash. On examination, his temperature is 38.3° C (101.0° F), and there is a prominent palpable reddish-brown rash on the buttocks and thighs. There is pain on motion of his knees and ankles, and mild diffuse abdominal tenderness. The stool is positive for occult blood. Laboratory Findings: Hemoglobin 11.0 g/dL Hematocrit 33% WBCs 14,500/mm; 85% segs, 3 15% lymphs, Platelets 345,000/mm 3 Prothrombin time 12 sec
  • 84. QUESTION 51 Which one of the following is the most likely diagnosis? ANSWER a) Henoch-Schönlein purpura b) Rocky Mountain spotted fever c) Acute iron ingestion d) Systemic onset juvenile rheumatoid arthritis
  • 85. QUESTION 52 Which one of the following drug classes is preferred for treating hypertension in patients who also have diabetes mellitus? ANSWER a) alpha-Blocking agents b) ACE inhibitors c) Centrally-acting sympatholytics d) Calcium channel blockers
  • 86. CASE REPORT A 76-year-old white male with a history of recurrent depression has recently become more depressed and developed psychotic features. His symptoms have not responded to antidepressants and antipsychotic agents, prescribed by his psychiatrist. The psychiatrist has recommended electroconvulsive therapy (ECT) for the patient. The patient’s family visits you to ask for your opinion and recommendations regarding ECT in this individual.
  • 87. QUESTION 53 In your consultation with this family, which one of the following would be accurate advice regarding ECT? ANSWER a) It has a low response rate b) It is efficacious and safe c) There is evidence that it predisposes to the development of dementia d) It causes irreversible short-term memory loss
  • 88. QUESTION 54 Metformin, which is normally used in the management of diabetes mellitus, has also been shown to have a beneficial effect in ANSWER a) hyperthyroidism b) polycystic ovary syndrome c) right ventricular hypertrophy d) osteoporosis
  • 89. CASE REPORT A 45-year-old white male presents with a 3-day history of new-onset acute upper abdominal pain radiating to the back. His pulse rate is 110 beats/min and other vital signs are normal. Bowel sounds are hypoactive and epigastric tenderness without guarding is present.
  • 90. QUESTION 55 Which one of the following is most accurate regarding laboratory testing for possible pancreatitis in this situation? ANSWER a) Serum amylase and lipase levels may be falsely low in renal failure b) Elevated serum triglyceride levels can cause falsely elevated serum amylase levels c) There is a direct correlation between serum amylase levels and the severity of the pancreatitis d) A serum amylase level that is three times normal is highly specific for pancreatitis
  • 91. QUESTION 56 You are evaluating a 28-year-old primigravida for an abnormal Papanicolaou (Pap) test. Which one of the following procedures would be contraindicated? ANSWER a) Endocervical curettage b) Human papillomavirus (HPV) testing c) Colposcopy d) Cervical staining
  • 92. QUESTION 57 Which one of the following should be withheld before the administration of intravenous contrast? ANSWER a) Insulin b) Metformin c) Rosiglitazone d) Glyburide
  • 93. CASE REPORT You are considering how useful a new treatment might be in preventing stroke. A well designed study is reported with 200 patients in the treated group and 200 patients in the untreated group. The study finds a 5-year risk of stroke of 3% in the treated group versus 5% in the untreated group.
  • 94. QUESTION 58 Assuming this study is valid and applicable to your patient population, how many patients would you have to treat for 5 years to prevent one stroke (number needed to treat, or NNT)? ANSWER a) 200 b) 25 c) 400 d) 50
  • 95. CASE REPORT Over the past year, a 27-year-old female has had marked feelings of anxiety, tension, and irritability during the week preceding most menstrual cycles, accompanied by extreme fatigue and insomnia. She has regularly missed several days of work each month because of fatigue. She has no previous history of any health or mental problems, and within a few days of the onset of her period she is back to normal.
  • 96. QUESTION 59 Which one of the following is true concerning this condition? ANSWER a) Alprazolam is an effective first-line agent for treatment of this condition b) This condition is a variation of a depressive disorder c) Neither biologic nor psychological factors play a part in this condition d) This problem can be effectively treated with serotonergic antidepressants
  • 97. CASE REPORT An 81-year-old retired electrical engineer whose wife is a diabetic was experimenting with his wife’s glucose meter and found that his glucose level was 198 mg/dL. He used her strips and lancets, and started his own log. After a week, he brings the log to you. His premeal glucose levels range from about 150 mg/dL to 250 mg/dL. A review of his medications shows none that would be likely to increase his glucose level. A physical examination does not suggest glucose intolerance secondary to a process other than diabetes. His hemoglobin A1c is 9.0%. Additional laboratory studies should be performed before prescribing
  • 98. QUESTION 60 which one of the following for this patient? ANWSER a) Metformin b) Glimepiride c) Miglitol d) Insulin NPH
  • 99. QUESTION 61 A 23-year-old gravida 3 para 1 at 28 weeks gestation whose blood type is O-negative is antibody positive (D antibody) on a routine 28-week screen. Which one of the following best describes the clinical significance of this finding? ANSWER a) The fetus HAS hemolytic disease and requires appropriate monitoring and treatment b) The current fetus is NOT at risk for hemolytic disease, but subsequent pregnancies may be at risk c) The fetus is AT RISK for hemolytic disease only if the biological father is Rh-negative d) The fetus is AT RISK for hemolytic disease only if the biological father is Rh-positive
  • 100. CASE REPORT A 5-year-old African-American male presents with behavior problems noted in the first 3 months of kindergarten. The mother explains that the child does not pay attention and often naps in class. He averages 10 hours of sleep nightly and is heard snoring frequently. The mother has a history of attention-deficit disorder and takes atomoxetine (Strattera). The boy’s examination is within normal limits except for his being in the 25th percentile for weight and having 3+ tonsillar enlargement.
  • 101. QUESTION 62 The most reasonable plan at this point would include which one of the following? ANSWER a) Polysomnography b) An electroencephalogram c) Methylphenidate d) Atomoxetine
  • 102. CASE REPORT A 16-year-old male is brought to your office by his mother for “stomachaches.” On review of systems, he also complains of headaches, occasional bedwetting, and trouble sleeping. His examination is within normal limits. His mother says that he is often in the nurse’s office at school, and doesn’t seem to have any friends. After some questions from you, he admits to being called names and teased at school.
  • 103. QUESTION 63 Which one of the following would be most appropriate? ANSWER a) Explain that these symptoms are a stress reaction and will lessen with time b) Explain that he must try to conform to be more popular c) Order a TSH level d) Explore whether his school counselor has a process to address this problem
  • 104. QUESTION 64 You are evaluating a 68-year-old male with obstructive urinary symptoms. Which one of the following medications may lead to falsely depressed levels of prostate-specific antigen (PSA)? ANSWER a) Tamsulosin b) Doxazosin c) Lycopene d) Finasteride
  • 105. QUESTION 65 The most appropriate initial treatment for scabies in an 8-year-old male is? ANSWER a) trimethoprim/sulfamethoxazole orally for 10 days b) 0.5% malathion lotion c) 5% permethrin cream d) 5% precipitated sulfur in petroleum
  • 106. CASE REPORT A 3-year-old female presents with urinary frequency, dysuria, and fever to 39.0° C (102.2° F). She denies nausea, vomiting, fever, and flank pain. There is no prior history of urinary infection and no family history of urinary tract abnormalities. Urethral catheterization reveals bacteriuria and a urine culture reveals >100,000 colony-forming units of Escherichia coli. She is started on appropriate antibiotic therapy.
  • 107. QUESTION 66 Evaluation to rule out anatomic abnormalities should include? ANSWER a) renal ultrasonography and voiding cystourethrography (VCUG) only if she has recurrent infections b) renal ultrasonography only if she has recurrent infections c) renal ultrasonography for this primary episode of infection d) renal ultrasonography and VCUG for this primary episode of infection
  • 108. CASE REPORT A slender 22-year-old female is concerned about a recent weight loss of 10 lb, frequent mild abdominal pain, and significant diarrhea of 2 months’ duration. Her physical examination is unremarkable, and laboratory studies reveal only a moderate microcytic, hypochromic anemia. Based on this presentation.
  • 109. QUESTION 67 which one of the following is the most likely diagnosis? ANSWER a) Villous adenoma b) Ulcerative colitis c) Infectious colitis d) Celiac disease
  • 110. QUESTION 68 A 73-year-old white male has severe COPD manifested by repeated hospital admissions, 30-step dyspnea on exertion, asthenia, and a resting pO of 58 mm Hg. Of the following, which intervention will most likely positively affect his survival? ANSWER a) Inhaled corticosteroids b) Beta -Agonists c) Oxygen supplementation d) Pulsed antibiotic therapy
  • 111. QUESTION 69 In patients with bipolar depression, monotherapy with which one of the following may trigger a manic episode? ANSWER a) Lithium b) Carbamazepine c) Lamotrigine d) Fluoxetine
  • 112. CASE REPORT A 30-year-old female presents with concerns about vaginal bleeding. She states that her menstrual periods have occurred at regular intervals of 28–30 days for the past 15 years, but recently bleeding has also occurred for a day or two in the middle of her cycle. This bleeding has been heavy enough to require the use of multiple pads.
  • 113. QUESTION 70 Which one of the following terms best describes her bleeding pattern? ANSWER a) Polymenorrhea b) Metrorrhagia c) Menometrorrhagia d) Acute emergent abnormal uterine bleeding
  • 114. QUESTION 71 A 25-year-old female presents with abdominal pain localized to the right lower quadrant. Which one of the following would be most helpful in diagnosing acute appendicitis? ANSWER a) Abdominal/pelvic ultrasonography b) A CBC c) Plain abdominal films d) Abdominal/pelvic C
  • 115. QUESTION 72 A 17-year-old white female presents with new-onset left-sided lower abdominal pain. Color flow Doppler ultrasonography, in addition to pelvic ultrasonography, would be most useful for evaluating? ANSWER a) pelvic inflammatory disease b) pelvic abscess c) adnexal torsion d) ruptured ovarian cyst
  • 116. QUESTION 73 A 30-year-old female presents with a vaginal discharge. On examination the discharge is homogeneous with a pH of 5.5, a positive whiff test, and many clue cells. Which one of the following findings in this patient is most sensitive for the diagnosis of bacterial vaginosis? ANSWER a) The character of the discharge b) The presence of clue cells c) The pH of the discharge d) The whiff test
  • 117. CASE REPORT A 25-year-old female at 36 weeks gestation presents for a routine prenatal visit. Her blood pressure is 118/78 mm Hg and her urine has no signs of protein or glucose. Her fundal height shows appropriate fetal size and she says that she feels well. On palpation of her legs, you note 2+ pitting edema bilaterally.
  • 118. QUESTION 74 Which one of the following is true regarding this patient’s condition? ANSWER a) Her leg swelling requires no further evaluation b) She most likely has deep venous thrombosis c) She most likely has preeclampsia d) You should order a 24-hr urine for protein
  • 119. QUESTION 75 Which one of the following is most likely to induce withdrawal symptoms if discontinued abruptly? ANSWER a) Divalproex b) Venlafaxine c) Fluoxetine d) Olanzapine
  • 120. CASE REPORT A 78-year-old male comes to your office with a 3-day history of pain in the right side of his chest. The pain is described as burning and intense. Two days ago he noted a rash at that site. Examination reveals groups of vesicles on an erythematous base in a T-5 dermatome distribution on the right.
  • 121. QUESTION 76 Which one of the following would be the most appropriate treatment to minimize the chance of post-herpetic neuralgia? ANSWER a) Carbamazepine b) Capsaicin c) Prednisone d) Famciclovir
  • 122. QUESTION 77 Which one of the following is the most common cause of bacterial diarrhea? ANSWER a) Shigella dysenteriae b) Escherichia coli O157:H7 c) Campylobacter jejuni d) Salmonella enterica
  • 123. CASE REPORT A 28-year-old gravida 2 para 1 at 32 weeks gestation presents with severe itching. She denies fever or vomiting. Her physical examination is remarkable for jaundice, but is otherwise benign. Laboratory studies reveal a normal CBC, normal platelets, normal glucose and serum creatinine levels, normal transaminase levels, and a bilirubin level of 4.0 mg/dL.
  • 124. QUESTION 78 Which one of the following is the most likely diagnosis? ANSWER a) Serum hCG levels should double every 2–3 days if the pregnancy is viable b) Painless bleeding excludes the diagnosis of ectopic pregnancy c) Laparoscopy should be performed to exclude ectopic pregnancy d) A serum progesterone level >25 ng/mL indicates that ectopic pregnancy is likely
  • 125. CASE REPORT A 35-year-old white female presents with a 6-month history of irregular menstrual bleeding. Before this problem began, her periods occurred every 30 days and lasted 5 days. Now they occur every 20 days and last for 10 days, and are heavier than they were previously. A physical examination reveals no obvious anatomic source of bleeding, and a Papanicolaou (Pap) test is normal. A pregnancy test is negative and a blood workup for organic causes of irregular menses is also negative. She takes no medications.
  • 126. QUESTION 79 Which one of the following would be most appropriate at this point? ANSWER a) Reassurance that the problem will resolve on its own b) Changing to a progesterone-only contraceptive c) Increasing the dosage of the oral contraceptive d) Pelvic ultrasonography
  • 127. QUESTION 80 Which one of the following is true regarding the risk of physical spouse abuse? ANSWER a) It increases with alcohol and substance abuse b) It decreases when a woman exits an abusive relationship c) It is higher among patients from racial minorities d) It increases as socioeconomic status rises
  • 128. QUESTION 81 Which one of the following causes rhinitis medicamentosa with prolonged use in the treatment of rhinitis? ANSWER a) Intranasal decongestants b) Intranasal antihistamines c) Intranasal mast cell stabilizers d) Leukotriene antagonists
  • 129. CASE REPORT An 83-year-old female presents to your office as a new patient. She recently moved to the area to be closer to her family. A history reveals that she has been in excellent health, has no complaints, and is on no medications except occasional acetaminophen for knee pain. She has never been in the hospital and has not had any operations. She says that she feels well. The examination is normal, with expected age- related changes, except that her blood pressure on three different readings averages 175/70 mm Hg. These readings are confirmed on a subsequent follow-up visit. In addition to lifestyle changes.
  • 130. QUESTION 82 which one of the following would be most appropriate for the initial management of this patient’s hypertension? ANSWER a) A thiazide diuretic b) An angiotensin receptor blocker c) An ACE inhibitor d) A beta-blocker
  • 131. QUESTION 83 Surgery for obesity would be most appropriate for which one of the following? ANSWER a) A 37-year-old African-American female who has failed multiple diets, has no coexisting medical problems, and has a BMI of 38 b) A 43-year-old African-American male with type 2 diabetes mellitus who has failed multiple diets and has a BMI of 38 c) A 31-year-old white male who has lost 50 lb on his diet and improved his sleep apnea, hypertension, and diabetes mellitus, but still has a BMI of 42 d) A 45-year-old white female with a history of depression and alcohol abuse who has failed multiple diets and has a BMI of 40
  • 132. CASE REPORT A 22-year-old gravida 2 para 1 presents to your office with a 1-day history of vaginal bleeding and abdominal pain. Her last menstrual period was 10 weeks ago, and she had a positive home pregnancy test 6 weeks ago. She denies any passage of clots. On pelvic examination, you note blood in the vaginal vault. The internal cervical os is open.
  • 133. QUESTION 84 Which one of the following best describes the patient’s current condition? ANSWER a) Completed abortion b) Threatened abortion c) Incomplete abortion d) Inevitable abortion
  • 134. CASE REPORT A 34-year-old white female visits your office with a chief complaint of pelvic pain that intensifies with her menstrual period. She has a history of pain during intercourse, which started in her mid-twenties and has gradually become worse. She reports recently missing some work during her menstrual period due to the pain. She has had two uneventful deliveries and the pain was absent during and after each pregnancy, but gradually returned. She and her husband do not wish to have any more children and her husband has had a vasectomy. The patient denies vaginal discharge or fever and a review of systems is negative. A complete physical examination is normal except for moderate nonspecific tenderness on pelvic examination. In addition, her uterus is moderately retroverted and has decreased mobility.
  • 135. QUESTION 85 Which one of the following would be the most appropriate initial step in the management of this patient? ANSWER a) A complete hysterectomy and bilateral oophorectomy b) Conjugated estrogens c) Depot medroxyprogesterone acetate d) contraceptives
  • 136. CASE REPORT A 62-year-old male presents for surgical clearance prior to transurethral resection of the prostate. His past history is significant for a pulmonary embolus after a cholecystectomy 15 years ago. His examination is unremarkable except that he is 23 kg (50 lb) overweight.
  • 137. QUESTION 86 The most appropriate recommendation to the urologist would be to? ANSWER a) place the patient on 650 mg of aspirin daily prior to surgery b) start warfarin after surgery with a goal INR of 1.5 c) start the patient on subcutaneous enoxaparin, 40 mg 1–2 hr prior to surgery and once a day after surgery d) cancel the surgery indefinitely
  • 138. CASE REPORT A 28-year-old white female sees you for preconception counseling. For the past 3 years she has been successfully treated with fluoxetine (Prozac) for depression, and she asks if she can continue taking it when she becomes pregnant. It is labeled by the FDA as category C for use in pregnancy.
  • 139. QUESTION 87 Which one of the following would you advise? ANSWER a) Animal studies do not indicate any risk to the fetus; there are no studies in women b) Animal studies demonstrate some risk to the fetus; there are no studies in women c) Controlled studies in women fail to demonstrate risk to the fetus; it is safe to continue it d) There is evidence of harm to the human fetus, and she should discontinue it
  • 140. QUESTION 88 Which one of the following is most predictive of increased perioperative cardiovascular events associated with noncardiac surgery in the elderly? ANSWER a) An age of 80 years b) Renal insufficiency (creatinine 2.0 mg/dL) c) Left bundle-branch block d) A history of previous stroke
  • 141. QUESTION 89 Which one of the following statements regarding varicoceles is true? ANSWER a) The incidence of varicoceles in adult males is <5% b) Varicoceles usually begin between 5 and 8 years of age c) A unilateral varicocele on the right side should be referred for further evaluation d) Most varicoceles are bilateral
  • 142. QUESTION 90 A 2-year-old white female is brought to your office by her parents, who are concerned about the child’s “flat feet.” On evaluation, the child’s feet are flat with weight-bearing, but with toe standing and with sitting the arch appears. You would? ANSWER a) reassure the parents b) recommend surgery c) recommend foot-stretching exercises d) recommend orthotics
  • 143. QUESTION 91 When a woman less than 50 years of age develops vulvar cancer, which one of the following associated conditions is most frequently present? ANSWER a) Lichen sclerosus b) Syphilis c) Human papillomavirus d) Lymphogranuloma venereum
  • 144. QUESTION 92 A 35-year-old white male with known long QT syndrome has a brief episode of syncope requiring cardiopulmonary resuscitation. Which one of the following is most likely responsible for this episode? ANSWER a) Asystole b) Atrial flutter with third degree block c) Sinus tachycardia d) Torsades de pointes
  • 145. QUESTION 93 Which one of the following is associated with a reduced risk of postmenopausal osteoporosis? ANSWER a) Diuretic use b) Cigarette smoking c) Low BMI d) Corticosteroid use
  • 146. QUESTION 94 A 59-year-old female with type 2 diabetes develops a 2×1-cm ulcer on the plantar aspect of her right foot. The ulcer is very deep and there is surrounding cellulitis. A plain film is normal. Which one of the following would be the imaging study of choice to rule out osteomyelitis in this patient? ANSWER a) A leukocyte scan b) A CT scan c) Angiography d) An MRI scan
  • 147. CASE REPORT • An 83-year-old female is admitted to the hospital with an exacerbation of her COPD. On the second hospital day she is clinically improved but is quite disoriented, experiencing visual hallucinations, agitation, and problems with recent memory and attention span. She is noted bynthe nursing staff to periodically fall asleep during conversation. Her previous medical history is notable for emphysema and hypertension, but there is no history of psychiatric problems. Her blood pressure is 140/82 mm Hg, pulse 88 beats/min, and oxygen saturation 98% on 2 L of nasal Oxygen.
  • 148. QUESTION 95 Which one of the following does this patient most likely have? ANSWER a) Schizophrenia b) Mania c) Acute depression d) Delirium
  • 149. QUESTION 96 A 31-year-old white female presents with her third stress fracture of a lower extremity in the past4 years. Her history and examination are otherwise unremarkable except for a controlled seizure disorder. The most likely cause of her bone problem is? ANSWER a) hypothyroidism b) Addison’s disease c) anticonvulsive medication d) osteogenesis imperfecta
  • 150. QUESTION 97 A 25-year-old female has unprotected intercourse and chooses to use Plan B (two 0.75 mg tablets of levonorgestrel, taken 12 hours apart) as a form of emergency contraception. Plan B has been shown to ANSWER a) prevent implantation, ovulation, and fertilization b) protect against sexually transmitted diseases c) be teratogenic to an already established pregnancy d) be effective only if used within 24 hours of unprotected intercourse
  • 151. QUESTION 98 The antimalarial agent mefloquine would be contraindicated in which one of the following patients? ANSWER a) A 50-year-old female with chronic renal failure and a creatinine clearance of 50 mg/min b) A 25-year-old female at 32 weeks gestation c) A 55-year-old male with COPD who uses inhaled albuterol and ipratropium bromide d) A 45-year-old male with epilepsy controlled with phenytoin.
  • 152. CASE REPORT Recent studies and meta-analyses have shown that supplementation with a particular vitamin may do more harm than good. This has resulted in a reversal in the recommendation for its use in patients with known coronary artery disease, and a recommendation for caution in its use in general for elderly patients.
  • 153. QUESTION 99 This is true of which one of the following? ANSWER a) Vitamin A b) Vitamin C c) Vitamin B 12 d) Vitamin E
  • 154. CASE REPORT In early February, you receive a call from your office nurse. Her 5-month-old daughter has been ill for several days. What started as a mild upper respiratory infection has progressed and she now has profuse rhinorrhea, a temperature of 100.2° F (37.9° C), and audible wheezing. In spite of an almost nonstop cough, she does not appear acutely ill.
  • 155. QUESTION 100 The organism responsible for this child’s illness is most likely to be? ANSWER a) Bordetella pertussis b) respiratory syncytial virus c) group B Streptococcus d) parainfluenza virus 3
  • 156. QUESTION 101 Of the following, which is the most frequent cause of seizures in the elderly? ANSWER a) Stroke b) Head trauma c) Dementia d) Alcohol withdrawal
  • 157. CASE REPORT A 4-week-old full-term male is brought to your office by his parents. They report that their child started vomiting just after his 1-week visit. The parents are concerned because they think the vomiting is worsening, occurring after every feeding, and “shooting across the room.” You note that the baby is afebrile, but has not gained any weight since birth.
  • 158. QUESTION 102 A 32-year-old gravida 3 para 2 is in labor at term following an uncomplicated prenatal course. As you deliver the fetal head it retracts against the perineum. Downward traction fails to free the anterior shoulder. The most appropriate course of action would be to? ANSWER a) have an assistant apply fundal pressure b) place the mother’s thighs on her abdomen c) apply increasingly strong downward traction to the fetal head d) deliberately fracture the clavicle of the fetus
  • 159. CASE REPORT A 14-year-old male is brought to your office by his mother to establish care. The patient has been diagnosed with asthma, but has not been on any medications for the past year. When questioned, he reports that his asthmatic symptoms occur daily and more than one night per week. On examination, he is found to have a peak expiratory flow of 75%.
  • 160. QUESTION 103 Based on these findings, the most accurate classification of this patient’s asthma is? ANSWER a) mild persistent b) moderate persistent c) severe persistent d) mild intermittent
  • 161. QUESTION 104 Which one of the following Papanicolaou (Pap) test results is most likely to indicate a cancerous lesion? ANSWER a) High-grade squamous intraepithelial lesion (HSIL) b) Atypical glandular cells not otherwise specified (AGC-NOS) c) Low-grade squamous intraepithelial lesion (LSIL) d) Atypical squamous cells cannot exclude high-grade intraepithelial lesion (ASC-H)
  • 162. CASE REPORT A 72-year-old female has stable but moderately severe COPD requiring 2 L of continuous oxygen. She plans to attend the college graduation of her first grandchild, and wants to fly to avoid a 12-hour car ride. Her PaO on room air is 55 mm Hg.
  • 163. QUESTION 105 • According to the Federal Air Regulations, she MUST? ANSWER a) find a form of transportation other than air travel b) bring a medical certificate from you certifying that she is cleared to fly without oxygen c) arrange through the airline for oxygen to be available on board and in the airports d) undergo preflight testing, including pulmonary function testing and high-altitude simulation testing, before she can be cleared to fly
  • 164. QUESTION 106 Which one of the following criteria is most likely to signify that an individual has bulimia nervosa rather than anorexia nervosa? ANSWER a) The absence of at least three consecutive menstrual cycles in a postmenarchal nonpregnant female b) Recurrent episodes of binge eating c) Intense fear of weight gain or becoming fat, even though underweight d) Denial of the seriousness of the current low body weight
  • 165. QUESTION 107 Painful ingrown toenails that display granulation tissue and lateral nail fold hypertrophy are best treated by? ANSWER a) removal of the entire nail b) cotton-wick elevation of the affected nail corner c) antibiotic therapy d) excision of the lateral nail plate combined with lateral matricectomy
  • 166. CASE REPORT A 36-year-old female makes an appointment because her husband of 12 years was just diagnosed with hepatitis C when he tried to become a blood donor for the first time. He recalls multiple blood transfusions following a motorcycle crash in 1988. His wife denies past liver disease, blood transfusions, and intravenous drug use. She has had no other sexual partners. The couple has three children.
  • 167. QUESTION 108 Which one of the following is the best advice about testing the wife and their three children? ANSWER a) No testing is required if her husband has normal liver enzyme levels b) No testing is required because tests have low sensitivity c) All family members should be tested because of possible household fecal-oral spread d) She should be offered testing because sexual transmission is possible
  • 168. QUESTION 109 You are evaluating a 5-month-old with fever, tachypnea, and mild respiratory distress in the emergency department. You hear mild basilar rales. The child does not appear toxic. Which one of the following tests would be the most appropriate as an initial study? ANSWER a) A C-reactive protein level b) A chest radiograph c) A CBC d) Oxygen saturation by pulse oximetry
  • 169. QUESTION 110 The elderly patient may process medications differently than a younger adult. Which one of the following is LEAST affected by aging? ANSWER a) Absorption b) Distribution c) Compliance d) Metabolism
  • 170. QUESTION 111 A 72-year-old Asian female is found to have asymptomatic gallstones on abdominal ultrasonography performed to evaluate an abdominal aortic aneurysm. Which one of the following would be the most appropriate management for the gallstones? ANSWER a) Lithotripsy b) Laparoscopic cholecystectomy c) Open cholecystectomy d) Observation
  • 171. • A 34-year-old female presents to you for preconception counseling regarding the management of her chronic hypertension. Her blood pressure has been well controlled on benazepril, 20 mg/day, without any side effects. The patient’s blood pressure was 145/95 mm Hg prior to beginning benazepril. She has been pregnant once before, and her physician switched her to methyldopa during that pregnancy, but she suffered from drowsiness and a dry mouth during much of that time. The pregnancy and delivery were otherwise uncomplicated. She has no history of diabetes mellitus, renal insufficiency, or asthma. She is a nonsmoker.
  • 172. QUESTION 112 • Which one of the following would you do when she becomes pregnant? ANSWER a) Switch to atenolol until after delivery b) Switch to long-acting nifedipine until after delivery c) Continue the benazepril through the pregnancy and delivery d) Discontinue the benazepril and monitor closely throughout the pregnancy for signs of preeclampsia or fetal growth restriction
  • 173. QUESTION 113 Which one of the following is the preferred treatment for patients with obsessive-compulsive disorder? ANSWER a) Valproic acid b) Alprazolam c) Fluoxetine d) Lithium carbonate
  • 174. QUESTION 114 When repairing a perineal laceration after a vaginal delivery, which one of the following suture materials decreases both wound dehiscence and postpartum perineal pain? ANSWER a) Polyglactin 910 (Vicryl) b) Silk c) Plain catgut d) Polypropylene
  • 175. QUESTION 115 A 40-year-old female is scheduled for a cholecystectomy and you wish to estimate her risk for postoperative bleeding. Which one of the following provides the most sensitive method for identifying her risk? ANSWER a) Prothrombin time (PT) b) Activated partial thromboplastin time (aPTT) c) Bleeding history d) Bleeding time
  • 176. QUESTION 116 A mother is nearing the end of maternity leave. She asks for your advice regarding breastfeeding her infant after she returns to work. Which one of the following would be accurate advice? ANSWER a) Breast milk should be stored in glass bottles b) Separation of breast milk indicates spoilage c) Breast milk should be thawed in boiling water d) Refreezing breast milk destroys some proteins
  • 177. QUESTION 117 Compared to anesthesia using only parenteral opioids, the use of epidural anesthesia in labor and delivery increases the rate of which one of the following? ANSWER a) Low Apgar scores (<7) b) Maternal low backache 3 months post delivery c) Prolonged second stage of labor d) Cesarean section
  • 178. CASE REPORT A 70-year-old white female presents with a pruritic rash of her sacrum that has occurred intermittently over the last 6 years. She reports that the area is always very tender just before the blister-like lesions erupt. She is otherwise in good health, and takes no medications. Her past medical history is unremarkable. You provide appropriate treatment for the condition.
  • 179. QUESTION 118 You should advise the patient to avoid which one of the following during future outbreaks? ANSWER a) Sexual contact b) Prolonged sitting c) Sun exposure d) Excessive intake of green, leafy vegetables
  • 180. • A 32-year-old female presents with bilateral pretibial tender, mildly red nodules 2–4 cm in diameter. A nodule that appeared earlier resolved, leaving a “bruised” area. She had a similar problem once when she was pregnant but it resolved spontaneously. Her medications include lovastatin (Mevacor) for hyperlipidemia and a low-dose oral contraceptive prescribed 5 months earlier. Her past history and a review of systems are otherwise unremarkable.
  • 181. QUESTION 119 The most appropriate next step would be to ANSWER a) order a serum creatine phosphokinase level b) discontinue her oral contraceptive c) obtain a cervical culture for gonorrhea d) discontinue lovastatin
  • 182. QUESTION 120 A 23-year-old female was recently diagnosed with bipolar disorder after experiencing her first episode of acute severe mania. Which one of the following would be appropriate initial maintenance therapy for her? ANSWER a) No medication unless she has a second severe episode b) Valproate c) Gabapentin d) Lorazepam
  • 183. CASE REPORT A 23-year-old sexually active female presents to your office with a 2-week history of vaginal discharge and mild coital discomfort. On physical examination, you note the presence of a mucopurulent vaginal discharge and cervical friability. She is afebrile and there are no other positive physical findings. No trichomonads or yeast is seen on vaginal preparations. Material for Chlamydia trachomatis– specific DNA testing is submitted and results will be available in 2 days.
  • 184. QUESTION 121 Which one of the following is true regarding appropriate management? ANSWER a) No treatment is indicated until laboratory results are known b) metronidazole given now will clear her discharge c) A 7-day course of doxycycline is superior to a 1-g dose of azithromycin d) The patient should be instructed to refrain from sexual intercourse until 7 days after initiating therapy
  • 185. QUESTION 122 Compared to children with ADHD, adults with ADHD? ANSWER a) tend to be less impulsive b) tend to be more hyperactive c) are less likely to complain of inattention difficulties d) are less likely to have corroboration of symptoms by family members
  • 186. QUESTION 123 A 33-year-old female requests combined oral contraceptive pills (OCPs) for birth control. Which one of the following would be a contraindication to prescribing OCPs for this patient? ANSWER a) A family history of ovarian cancer b) A history of controlled hypertension c) A history of hepatitis C infection with no liver disease d) A history of thromboembolic disease
  • 187. QUESTION 124 Routine blood tests frequently reveal elevated calcium levels. When this elevation is associated with elevated parathyroid hormone levels, which one of the following is an indication for parathyroid surgery? ANSWER a) Concurrent hyperthyroidism b) Increased bone density c) Age >50 d) Kidney stones
  • 188. QUESTION 125 A 21-year-old female had a pelvic examination and a normal Papanicolaou (Pap) test 3 months ago. She is now being treated for chlamydial cervicitis with azithromycin, 1 g in a single dose. When should she next have a test for Chlamydia? ANSWER a) Never, unless she is symptomatic or has a suspected exposure to Chlamydia at some point in the future b) 3–4 months c) 1–2 weeks d) 9 months (at her next routine pelvic examination)
  • 189. QUESTION 126 A 10-year-old male develops a fever 3 months after returning to the U.S. following a year in the tropics. He was treated with the proper immunizations and given appropriate prophylactic medications before he went abroad with his parents. Which one of the following tropical illnesses should be considered in the differential diagnosis? ANSWER a) Typhoid fever b) Dengue fever c) Plague d) Malaria
  • 190. CASE REPORT A mother brings her 12-month-old son to your clinic, concerned that he is repeatedly banging his head against the floor, wall, or crib. She reports that this behavior began about 2 months ago. It now occurs several times per week, and at times is incited when the child is frustrated with a toy or when he does not get what he wants from his parents. The mother notes that she is sometimes awakened at night by the sound of her son rhythmically banging his head against the rail of hiscrib. Physical examination reveals a normal child with some soft-tissue swelling of the forehead, but no broken skin, ecchymosis, or signs of bony damage. Developmental milestones and growth have been normal, and the child is not on any medications.
  • 191. QUESTION 127 Children with this presentation are most likely to have which one of the following? ANSWER a) A skin laceration or skull fracture b) An eventual diagnosis of Lesch-Nyhan syndrome c) Extinction of this habit by age 3 d) Future cognitive delay when compared with children without this habit
  • 192. CASE REPORT A 4-year-old white female is brought to your office by her mother, who reports that the child recently developed a foul-smelling vaginal discharge. After an appropriate history and general examination, you determine that a genital examination is necessary.
  • 193. QUESTION 128 Which one of the following positions is most likely to allow for visualization of the child’s vagina and cervix without instrumentation? ANSWER a) The knee-chest position on an examination table b) Trendelenburg’s position on an examination table c) The left lateral decubitus position on an examination table d) Supine in the mother’s lap
  • 194. QUESTION 129 Which one of the following has been shown to transmit Salmonella infections to humans? ANSWER a) Iguanas b) Guinea pigs c) Civet cats d) Rabbits
  • 195. QUESTION 130 The likelihood of postpartum depression is increased by which one of the following? ANSWER a) A prior history of depression b) Bottle feeding c) Low educational level d) Cesarean delivery
  • 196. QUESTION A 31-year-old female presents with her first outbreak of genital herpes. She has been married for 4 years and says she has not had sexual relations with anyone other than her husband since they started dating.
  • 197. QUESTION 131 When counseling this patient, which one of the following would be appropriate advice? ANSWER a) Suppressive therapy can decrease transmission to sexual partners b) Daily application of topical acyclovir is effective for suppressant therapy c) If type-specific serologies are negative, genital herpes can be ruled out d) Because the average incubation period for herpes is 3 months, her husband has most likely had another sexual partner in the past few months
  • 198. CASE REPORT A 32-year-old female is concerned about ovarian cancer. She has no symptoms at this time. However, she has a close friend who was recently diagnosed with ovarian cancer at an advanced stage. This friend told the patient that a “simple blood test” for CA-125 could detect ovarian cancer at a curable stage.
  • 199. QUESTION 132 As part of your discussion, you inform the patient that which one of the following is true? ANSWER a) A high serum CA-125 level is not associated with ovarian cancer b) Most consensus opinions recommend performing this test for average-risk women c) This test should not be ordered due to its high false- negative rate d) Detecting ovarian cancer at an earlier stage using serum CA-125 has not been shown to reduce mortality
  • 200. QUESTION 133 Ehrlichiosis may result from exposure to? ANSWER a) freshwater snails b) fleas c) ticks d) rats
  • 201. QUESTION 134 The best method of influenza prophylaxis in high-risk patients is administration of? ANSWER a) oral amantadine daily during the influenza season b) influenza vaccine intramuscularly prior to the influenza season c) oral rimantadine daily during the influenza season d) oral oseltamivir daily during the influenza season
  • 202. QUESTION 135 A 75-year-old male is brought in by his family due to concerns about memory loss. Which one of the following is best for determining whether or not he has dementia? ANSWER a) A Mini-Mental State Examination (MMSE) b) Non–contrast-enhanced CT of the head c) An electroencephalogram d) MRI of the head
  • 203. QUESTION 136 Of the following, which one has the greatest potential to spread via human-to-human transmission? ANSWER a) Pneumonic plague b) Botulism c) Typhoidal tularemia d) Coxiella burnetii (Q fever)
  • 204. QUESTION 137 The mother of a 3-year-old male is concerned that he doesn’t like being held, doesn’t interact much with other children, and rarely smiles. Of the following, which feature would be most helpful in distinguishing Asperger’s syndrome from autism in this patient? ANSWER a) Repetitive fine motor mannerisms b) Delayed gross motor development c) Normal language development d) Preoccupation with parts of objects
  • 205. CASE REPORT A 76-year-old male is brought to your office by his son. The patient complains of dizziness that has slowly been worsening over the past year. His description is vague, but he says that he notices the dizziness when he tries to walk. The review of systems is normal, except for increasing problems with his “prostate” in the last 6 months, with dribbling and accidents at times. The patient admits to “going more often,” and sometimes “without warning.” The son states that his father seems more forgetful, slower of speech, and not as “full of life” as he used to be. A neurologic examination reveals the patient to be oriented × 3, with a somewhat flat affect and a wide- based, slow, shuffling gait. The examination is otherwise normal. His Mini-Mental State Examination (MMSE) score is 22 out of 30. No tremor is noted. A CBC, creatinine level, TSH level, vitamin B 12 level, and VDRL are all normal.
  • 206. QUESTION 138 Which one of the following should you do next? ANSWER a) Order a brainstem auditory evoked potential b) Prescribe sertraline (Zoloft) c) Order physical therapy d) Order a brain MRI
  • 207. QUESTION 139 The most appropriate initial pharmacologic treatment of panic disorder is? ANSWER a) valproic acid (Depakene) b) lithium c) an SSRI d) a tricyclic antidepressant
  • 208. CASE REPORT A 24-year-old worker at a local day-care center has not had a menstrual period in over 2 months. A home pregnancy test is positive and is confirmed in your office. She is concerned because she was recently exposed to a child who was diagnosed with rubella. A serum rubella IgG antibody test is negative.
  • 209. QUESTION 140 Which one of the following would be the best course of action? ANSWER a) Order level III ultrasonography to screen for congenital anomalies b) Repeat the serologic testing in 2–3 weeks c) Administer rubella immune globulin d) Tell the patient she is immune to rubella and is not at risk
  • 210. CASE REPORT A 23-year-old male with Down syndrome is brought to your office by his parents. The patient has had a low- grade fever (approximately 100° F) for 7 days. The fever is not associated with rhinorrhea, sore throat, cough, dysuria, or shortness of breath. The parents note that their son has complained of aches in various places, and has not been eating well for several weeks. The physical examination is remarkable for swollen lymph glands in multiple places, pale conjunctivae, and an 8-lb weight loss since his last visit 1 month ago.
  • 211. QUESTION 141 Which one of the following is most consistent with this patient’s symptoms? ANSWER a) Urinary tract infection b) Atypical pneumonia c) Acute myeloid leukemia d) Hodgkin’s lymphoma
  • 212. QUESTION 142 Which one of the following is true regarding Osgood- Schlatter disease? ANSWER a) Complete symptom resolution usually occurs in 18–24 months b) Effusions are often present on examination and knee range of motion is limited c) Most cases are triggered by a single traumatic event d) Pain is localized to the inferior patellar pole at the origin of the patellar tendon
  • 213. QUESTION 143 Which one of the following is an indicator of low risk for streptococcal infection in a patient with acute pharyngitis? ANSWER a) Soft palate petechiae b) Cough c) Anterior cervical lymphadenopathy d) Fever
  • 214. QUESTION 144 Your community recently experienced an outbreak of infectious diarrheal illness due to the protozoan Cyrptosporidium, a chlorine resistent organism. A reporter from the local newspaper asks you if there are other chlorine-resitant fecal organisms that could contaminate public drinking water. You would tell the reporter that such organisms include. 15/20 missed ANSWER a) Escherichia Coli b) Giardia Lamblia c) Rotavirus d) Campylobacter jejuni
  • 215. CASE REPORT A 32-year-old white female comes to your office complaining of dysuria. She denies fever , back pain, and urinary frequency. She appears to be well otherwise, and has a normal abdominal examination. A clean-catch urinalysis shows 15-20 WBCs/hpf and a dipstick test for leukocyte esterase is positive. You send a urine sample for culture and start the patient on nitrofurantoin (Macrodantin), as she is allergic to sulfa. Three days later, the patient returns with persistent dysuria despite taking the medication as prescribed. Her urine culture has returned with no growth. A pelvic examination is normal, and the rest of the physical examination is unchanged. A wet prep is normal and tests for sexually transmitted diseases are pending.
  • 216. QUESTION 145 Which one of the following antibiotics is most appropriate for this patient now? ANSWER a) Metronidazole b) Doxycycline c) Cephalexin d) Ciprofloxacin
  • 217. QUESTION 146 In which one of the following would misoprostol be appropriate? ANSWER a) A primigravida at term whose cervix is dilated to 7 cm and whose frequency of contractions has slowed b) A primigravida with a long, thick cervix at term who has a previous history of myomectomy c) A diabetic primigravida with a long, thick cervix at 38 weeks gestation for whom induction is indicated d) A gravida 2 para 1 with a long, thick cervix at term who is interested in vaginal birth after a cesarean section
  • 218. QUESTION 147 Which one of the following therapeutic interventions is useful in the treatment of croup, but is not an accepted treatment for bronchiolitis? ANSWER a) Oral dexamethasone b) Extracorporeal membrane oxygenation c) Nebulized racemic epinephrine d) Oxygen via the blow-by method
  • 219. CASE REPORT Your office nurse reports an accidental needlestick with a used venipuncture needle. She had drawn blood from a 35-year-old white male, who is a new patient undergoing evaluation for hypertension and hyperlipidemia. He has no evidence of other health problems and no risk factors for HIV. Both the nurse and the patient are found to be HIV negative.
  • 220. QUESTION 148 Which one of the following would be the most appropriate advice? ANSWER a) No further testing for HIV is necessary b) She should begin treatment with prophylactic antiretroviral therapy within the next 48 hours c) She should refrain from unprotected sexual contact with her husband until another negative test 6 weeks after the exposure d) She should have repeat HIV testing at 6, 12, 24, and 52 weeks after the exposure
  • 221. QUESTION 149 Which one of the following is consistent with current recommendations? ANSWER a) Cervical cytologic screening should be done yearly prior to the age of 30, with either a conventional Pap smear or liquid-based cytology b) Cervical cancer screening should begin no later than 18 years of age in all women c) Cervical cancer screening with Papanicolaou (Pap) smears should begin within 1 year after the onset of vaginal intercourse d) Screening with vaginal cytology is not indicated in women who have had a total hysterectomy for benign gynecologic disease
  • 222. QUESTION 150 The predominant symptom associated with dysthymic disorder is? ANSWER a) loosening of associations b) delusions c) incoherence d) depression
  • 223. QUESTION 151 Which one of the following is a common early side effect of fluoxetine? ANSWER a) Skin rash b) Orthostatic hypotension c) Constipation d) Loss of appetite
  • 224. CASE REPORT You see a 17 year old white female who has recently become sexually active. She requests oral contraceptives and you perform a brief evaluation, including blood pressure measurment. A Pregnancy test is negative. She is resistant to further evaluation unless it is necessary.
  • 225. QUESTION 152 In addition to appropriate counseling, which one of the following should be done before prescribing oral contraceptives? 17/20 missed. ANSWER a) No further evaluation at this visit unless indicated by history. b) A pelvic examination and Papanicolaou test c) Screening for sexully transmitted diseases. d) A breast exam.
  • 226. CASE REPORT You diagnose depression in a 27 year old whit male and prescribe buproprion. He returns for a follow up visit 4 weeks later. He now reports less despondency, but says he has developed severe insomnia. He also report that his mind "races" all the time, and that he has recieved two speeding tickets in the past 2 weeks
  • 227. QUESTION 153 Which one of the following would be appropriate now? ANSWER a) Add Valproate b) Add imipramine at bedtime c) Substitute sertraline for the buproprion d) Add Alprazolam
  • 228. CASE REPORT A 73-year-old white female presents to the office with her daughter. She complains of a several-year history of intermittent involuntary loss of urine which is gradually worsening. She frequently loses small amounts of urine when she rises from a low chair, and greater amounts if she coughs, sneezes, or laughs. She has taken to wearing absorbent undergarments and admits to significant embarrassment and limitation of activities as a result of her problem. Her daughter states that the patient has gradually withdrawn from her usual social contacts. Physical examination reveals a well-appearing elderly female. Her pelvic examination shows mucosal atrophy and a mild cystocele. With a full bladder, she loses a few ml of urine into a gauze pad when standing and coughing. Her post- void residual is normal. Her physical examination is otherwise normal for her age, and her urinalysis is normal.
  • 229. QUESTION 154 If nonpharmacologic interventions are not completely successful in the management of this patient's problem, which one of the following medications is most likely to help? 19/20 missed. ANSWER a) Bethanechol b) Hycosamine c) Flavoxate d) Pseudoephedrine
  • 230. QUESTION 155 Which one of the following is the medical treatment of choice for acute delirium in the intensive care unit? ANSWER a) Intravenous droperidol every 6-8 hours b) Intravenous haloperidol in increasing doses every 30 minutes as needed c) Intramuscular diphenhydramine d) Intramuscular chlorpromazine
  • 231. QUESTION 156 Screening for Chlamydia trachomatis infection should be performed on which one of the following asymptomatic patients who present for routine evaluation? ANSWER a) A 20-year-old female who uses birth control pills for contraception and has a new sexual partner b) A 22-year-old male with multiple sexual partners who uses condoms, but not consistently c) A 16-year-old female who states that she is not sexually active d) A 28-year-old female who is married, has no history of sexually transmitted disease, and uses the rhythm method to avoid pregnancy
  • 232. QUESTION 157 The most common hospital errors associated with preventable adverse drug effects are in the stage of 16/20 missed? ANSWER a) transcription b) dispensing c) administration d) ordering
  • 233. CASE REPORT A 73-year-old white female complains of difficulty reading for the past several months. Examination reveals 20/100 near vision bilaterally, and normal visual fields by confrontation. An ophthalmoscopic examination reveals only small yellow lesions clustered in the retina.
  • 234. QUESTION 158 The most likely diagnosis is? ANSWER a) glaucoma b) retinal detachment c) cataract d) macular degeneration
  • 235. QUESTION 159 The most effective way to diagnose chronic alcoholism is to? ANSWER a) ask the patient directly if he/she is an alcoholic b) confront the patient when he/she is intoxicated c) obtain a careful history of alcohol intake from the patient d) inquire about problems resulting from drinking
  • 236. QUESTION 160 Compared with younger adults, healthy older adults? ANSWER a) spend fewer hours in stages of deep sleep b) fall asleep more quickly at bedtime c) spend less time awake in bed d) awaken less frequently during the night
  • 237. QUESTION 161 Which one of the following is more likely to occur with glipizide than with metformin? ANSWER a) Lactic acidosis b) Gastrointestinal distress c) Hypoglycemia d) Weightloss
  • 238. QUESTION 162 While examining a male infant during a routine well child visit, you palpate only one testicle within the scrotum. Which one of the following is true concerning this problem? ANSWER a) The risk of malignancy in the undescended testis is several times higher than in the general population b) Surgery should be postponed until 3 years of age to allow for spontaneous descent c) Orchiopexy with antecedent hormonal therapy (LH- RH, hCG) is the currently recommended treatment d) In spite of orchiopexy, more than 50% of patients with a history of an undescended testis are infertile as adults
  • 239. CASE REPORT A 21-year-old white female presents to the emergency department with a history consistent with a lateral ankle sprain that occurred 2 hours ago while she was playing softball. She complains of pain over the distal anterior talofibular ligament, but is able to bear weight. There is mild swelling, mild black and blue discoloration, and moderate tenderness to palpation over the insertion of the anterior talofibular ligament, but the malleoli are nontender to palpation.
  • 240. QUESTION 163 Which one of the following statements is true regarding the management of this case? 13/20 missed. ANSWER a) Anteroposterior, lateral, and 30° internal oblique (mortise view) radiographs should be done to rule out fracture b) For best results, functional rehabilitation should begin within the first 24 hours after injury c) Stress radiographs will be needed to rule out a major partial or complete ligamentous tear d) Early range-of-motion exercises should be initiated to maintain flexibility
  • 241. QUESTION 164 In evaluating an adult with anemia, which one of the following findings most reliably indicates a diagnosis of iron deficiency anemia? ANSWER a) Microcytosis b) Low serum ferritin c) Low total iron-binding capacity d) Hypochromia
  • 242. QUESTION 165 During a routine well child examination of a 9-month- old female, you discover labial adhesions with a 5- mm opening anteriorly. Which one of the following statements is true concerning this condition? ANSWER a) It is rarely associated with urinary tract infections b) The labia majora are most likely affected c) It is rarely found in neonates d) Oral estrogens are an accepted alternate treatment
  • 243. CASE REPORT A 37-year-old gravida 6 para 5 is given oxytocin to induce delivery at 41 weeks gestation. Her prenatal course is significant for chronic hypertension. She delivers a 4020-g (8 lb 14 oz) baby. Soon after delivery of the placenta, she begins to have excessive vaginal bleeding.
  • 244. QUESTION 166 Which one of the following would you do initially? ANSWER a) Chromosome analysis b) A TSH level c) Phenylketonuria screening d) A serum creatine kinase level
  • 245. QUESTION 167 Of the following antidepressants, which one is LEAST likely to cause drug interactions? ANSWER a) Citalopram b) Paroxetine c) Fluoxetine d) Mirtazapine
  • 246. QUESTION 168 When advising women about the need for breast cancer screening, which one of the following statements is accurate regarding risk factors and the need for screening? ANSWER a) Hereditary breast cancer is more common in postmenopausal women b) Having several second degree relatives with breast cancer carries the same risk as having one first degree relative with breast cancer c) Most women with breast cancer have no identifiable risk factors d) When hereditary breast cancer strikes several generations of a family, women descended from a male family member are not at increased risk
  • 247. QUESTION 169 Of the following, the INITIAL treatment of choice in the management of severe hypertension during pregnancy is? ANSWER a) labetalol intravenously b) reserpine intramuscularly c) nifedipine sublingually d) enalapril intravenously
  • 248. QUESTION 170 A 35-year-old female complains that her nose is too large, even after having cosmetic surgery on her nose three times. She has a minimal social life because of her concern about the appearance of her nose. A physical examination, including her appearance, is normal. What is the most likely diagnosis? ANSWER a) Body dysmorphic disorder b) Social phobia c) Obsessive-compulsive disorder d) Delusional disorder
  • 249. QUESTION 171 A 39-year-old white male comes to your office with severe pain, fluctuance, erythema, and, tenderness localized over the pad of the distal long finger. The most likely diagnosis is? ANSWER a) digital cellulitis b) a felon (whitlow) c) a paronychia d) an eponychial abscess
  • 250. CASE REPORT You see a 24-year-old obstetric patient late in her third trimester for mild dysuria and urinary urgency. Microscopic examination of the urinary sediment is notable for bacteria and you make a presumptive diagnosis of cystitis.
  • 251. QUESTION 172 Which one of the following antibiotics would be CONTRAINDICATED? ANSWER a) Trimethoprim/sulfamethoxazole b) Cephalexin c) Nitrofurantoin d) Amoxicillin-clavulanate
  • 252. QUESTION 173 The antiphospholipid syndrome in women is commonly associated with ANSWER a) recurrent abortion b) secondary amenorrhea c) metrorrhagia d) dysmenorrhea
  • 253. QUESTION 174 Which one of the following statements is true regarding the association between oral contraceptive use and the risk of breast cancer? ANSWER a) The longer the duration of oral contraceptive use, the greater the risk of breast cancer b) The risk of breast cancer is directly correlated with a higher estrogen content in the oral contraceptive c) The risk of breast cancer in women who use oral contraceptives is linked to a family history of breast cancer d) There is no significant difference in the risk of breast cancer between women who have used oral contraception and those who have not
  • 254. CASE REPORT A 67-year-old white male sees you for a new patient visit. He is asymptomatic and has not seen a doctor in 10 years. He does not smoke or drink and takes no medication. He says he has a history of "mild high blood pressure" but has never been treated for this. His blood pressure today is 180/90 mm Hg. He has a decreased arteriovenous ratio on funduscopic examination, his point of maximal intensity is displaced laterally, and he has decreased pedal pulses.
  • 255. QUESTION 175 The most appropriate management at this point would be to? ANSWER a) perform two blood pressure measurements 1 week apart to establish the diagnosis of hypertension b) prescribe a diuretic c) order a laboratory workup to rule out causes of secondary hypertension d) order ambulatory blood pressure monitoring
  • 256. QUESTION 176 You see a 90-year-old male with a 5-year history of progressive hearing loss. The most common type of hearing loss at this age affects? ANSWER a) all frequencies roughly the same b) predominantly high frequencies c) predominantly low frequencies d) predominantly mid-frequencies
  • 257. QUESTION 177 A 24-year-old white female presents to the office with a 6-month history of abdominal pain. A physical examination, including pelvic and rectal examinations, is normal. Which one of the following would indicate a need for further evaluation? ANSWER a) Changes in stool consistency from loose and watery to constipation b) Worsening of symptoms at night c) Passage of mucus with bowel movements d) Abdominal bloating
  • 258. QUESTION 178 Which of the following is the most reliable clinical symptom of uterine rupture? 14/20 missed. ANSWER a) Vaginal bleeding b) Fetal distress c) Sudden tearing uterine pain d) Loss of uterine tone
  • 259. QUESTION 179 Which one of the following is a risk factor for preterm birth? ANSWER a) High prepregnancy weight b) Prior preterm delivery c) Smoking d) First-trimester bleeding
  • 260. QUESTION 180 The hospital nursery reports that a 24-hour-old male has developed "acne" confined to his nose and cheeks. Your examination confirms the presence of acneiform lesions,including papules. Which one of the following would be most appropriate? ANSWER a) Clindamycin topically b) Boric acid rinses c) No treatment d) An emollient lotion
  • 261. QUESTION 181 A 38-year-old alcoholic male has successfully completed outpatient alcohol detoxification and has plans to participate in Alcoholics Anonymous. Which one of the following pharmacologic agents can aid in relapse prevention? ANSWER a) Naloxone b) Bupropion c) Naltrexone d) Mirtazapine
  • 262. CASE REPORT An 85-year-old white male is brought to you for the first time by his son. The father has recently seen a neurologist who performed a workup for dementia and diagnosed moderate Alzheimer's disease.
  • 263. QUESTION 182 Which one of the following is true regarding the use of a cholinesterase inhibitor in this patient? ANSWER a) Agitation is often intensified by these agents b) If the patient has a vascular dementia rather than Alzheimer's dementia the drug will not be useful c) Nursing-home placement may be delayed a year or longer d) Memory is likely to improve significantly
  • 264. QUESTION 183 In a 27-year-old white female with irregular menstrual cycles and infertility , which one of the following would be more indicative of Cushing , s syndrome rather than the more common polycystic ovarian syndrome? ANSWER a) Acne b) Hirsutism c) Acanthosis nigricans d) Easy bruising
  • 265. QUESTION 184 You diagnose attention-deficit/hyperactivity disorder in a 10- year-old male, and the parents ask about drug treatment. Which one of the following would you tell the parents with regard to potential side effects of drug treatment? ANSWER a) The effect on eventual adult height is minimal in patients treated with methylphenidate or dextroamphetamine b) Methylphenidate has significantly fewer side effects than dextroamphetanrine c) Although anorexia may occur, it will not lead to substantial weight loss d) Methylphenidate may precipitate seizures
  • 266. CASE REPORT A 60-year-old female has been on conjugated equine estrogens/medroxyprogesterone (Prempro) since she went through menopause at age 52. She still has her uterus and ovaries. She is having no side effects that she is aware of and is experiencing no vaginal bleeding. She is worried about the health effects of her hormone replacement therapy and asks your advice about risks versus benefits.
  • 267. QUESTION 185 Which one of the following would be accurate advice regarding these risks and benefits? ANSWER a) The incidence of myocardial infarction is decreased b) The incidence of pulmonary embolus is decreased c) The incidence of colorectal cancer is increased d) The incidence of breast cancer is increased
  • 268. QUESTION 186 Which on of the following is the most effective initial treatment of head lice in an 8 year old child? ANSWER a) Head Shaving b) Malathion c) Wet combing every 4 days, to continue for 2 weks after any louse is found d) Lindane
  • 269. QUESTION 187 Which one of the following is associated with an increased risk of developing Alzheimer's disease? ANSWER a) A low-fiber diet b) A positive homozygous genotype for apolipoprotein E4 c) Elevated serum aluminum d) Decreased serum B-carotene
  • 270. QUESTION 188 Which one of the following is true regarding testicular torsion? ANSWER a) It is most common in boys under the age of 10 b) Testicular scanning is necessary prior to surgical exploration in order to document loss of blood flow to the testis c) Testicular torsion is often preceded by a communicating hydrocele d) Surgical exploration and detorsion must be done within 6 hours of the onset in order to preserve testicular function
  • 271. QUESTION 189 Which one of the following can contribute to serum calcium elevation? ANSWER a) Hydrochlorothiazide b) Enalapril c) Verapamil d) Allopurinol
  • 272. QUESTION 190 Which one of the following intravenous antibiotic regimens is most appropriate for the treatment of postpartum endometritis? ANSWER a) Ciprofloxacin plus doxycycline. b) Gentamicin plus methicillin c) Ciprofloxacin plus nafcillin d) Gentamicin plus clindamycin
  • 273. QUESTION 191 Female victims of domestic assault? ANSWER a) seldom spontaneously offer accounts of partner abuse b) usually seek medical care for their injuries c) are less likely to be abused during pregnancy d) usually resent unsolicited questions about abuse
  • 274. CASE REPORT To establish a reference range for a particular blood test in pregnant teenagers, your laboratory has plotted measurements for a large cohort of healthy subjects. The laboratory finds that these measurements fit a Gaussian distribution, and sets its reference range to include measurements within two standard deviations above and below the mean.
  • 275. QUESTION 192 What is the approximate probability that a healthy person will have a test result that falls outside the laboratory reference range? ANSWER a) 10% b) 0.01% c) 0.1% d) 5%
  • 276. QUESTION 193 If an abnormally high maternal serum alpha fetoprotein level is found at 16-18 weeks gestation, the next step is? ANSWER a) repeating the measurement of the maternal serum alpha fetoprotein at 24 weeks gestation. b) amniocentesis for amniotic fluid alpha fetoprotein, acetylcholinesterace, and karyotyping. c) recommending termination of the pregnancy. d) high resolution ultrasonography